Download as pdf or txt
Download as pdf or txt
You are on page 1of 40

GYNECOLOGY SAMPLEX

DRAVERGONZ 2022

QUIZ 1: FERTILIZATION, EMBRYOGENESIS, 7. The stage at which the oocytes are arrested
GENETICS, REPRODUCTIVE ENDO, until ovulation?
CONGENITAL ABNORMALITIES OF FEMALE a. Prophase I
REPRODUCTIVE TRACT b. Metaphase I
c. Meiosis II
1. The ff are the steps involved in implantation, d. NOTA
except:
a. Apposition 8. The round ligament is derived from?
b. Invasion a. Gubernaculum
c. Attachment b. Urogenital sinus
d. NOTA c. Medulla
d. NOTA
2. At what stage does the embryo enters the
uterine cavity? 9. Fertilization occurs in what part of the
a. Blast fallopian tube?
b. Zygote a. Ampulla
c. Morula b. Fimbriae
d. NOTA c. Cornua
d. AOTA
3. Where does blood formation occur first
during the second month of gestation? 10. What is the last fetal epithelium to develop?
a. Liver a. Ectoderm
b. Spleen b. Mesoderm
c. Bone Marrow c. Endoderm
d. Lymph nodes d. NOTA

4. What functions as the fetal kidney? 11. The ff arise from the urogenital sinus?
a. Pronephros a. Appendix vesiculosa
b. Metanephros b. Ureter
c. Mesonephros c. Greater vestibular gland
d. NOTA d. Duct of epididymis

5. Which hormone is responsible for 12. The ff is responsible for the development of
decidualization? testes
a. Progesterone a. SRY gene of Y chromosome
b. Estrogen b. XSR gene
c. Both A and B c. STR gene
d. NOTA d. NOTA

6. The ff is suppressed by the AMH/MIS? 13. The ff are nitrogen base purines
a. Wolffian duct a. Thymine
b. Mesonephric duct b. Adenine
c. Uterovaginal primordium c. Thymidine
d. NOTA d. Cytosine

GYNE TRANS TEAM Page 1 of 16


GYNECOLOGY SAMPLEX
DRAVERGONZ 2022

14. The principles of X inactivation involves c. 22q11.2


a. One X chromosome is inactivated d. NOTA
randomly in early male embryo
b. Either paternal or maternal X 21. Most common cause of labial fusion
chromosome is randomly chosen a. Exposure to exogenous androgens
c. All descendants will have different b. Congenital adrenal hyperplasia
inactive X chromosome c. 11-hydroxylas deficiency
d. NOTA d. 3B-hydroxysteroid dehydrogenase
deficiency
15. The ff is inherited as autosomal recessive
a. Friedrich Ataxia 22. A 16-yr old student consulted her physician
b. Fragile X due to absence of menses. She also
c. NOTA claimed to have been having cyclical pelvic
d. Both pain for the past 2 months. On PE, there
was a slight bulging membrane 3 cm
16. The ff are associated with live births and superior to the introitus. What’s the most
advanced maternal age probable diagnosis?
a. Trisomy 21 a. Imperforate hymen
b. Trisomy 13 b. Vaginal agenesis
c. Trisomy 18 c. Transverse vaginal septum
d. AOTA d. Mayer-Rokitansky-Kuster-Hauser
syndrome
17. The ff is the karyotype of Klinefelter
Syndrome 23. Saddle-shaped uterine fundus on
a. 47, XXX hysterosalpingography
b. 47, XXY a. Arcuate uterus
c. 47, XYY b. Uterus didelphys
d. NOTA c. Bicornuate uterus
d. DES-related uterine anomaly
18. Maternal AFP is used for detection of neural
tube defects between? 24. First-line treatment of vaginal agenesis
a. 12-14 wks AOG a. McIndoe vaginoplasty
b. 18-19 wks AOG b. Progressive vaginal dilatation
c. NOTA – (bet 16-18wks AOG) c. Vechietta procedure
d. AOTA d. Kinderman vaginoplasty

19. Syndrome due to terminal deletion 25. Accdg to the American Fertility Society as to
a. Patau Classification of Mullerian Abnormalities,
b. Cri du chat septate uterus belongs to
c. Turners a. Class II
d. NOTA b. Class V
c. Class III
20. Cat’s eye is a rare condition that involves d. Class VI
duplication of the following?
a. 22q11 26. Characteristics of Mayer-Rokitansky-Kuster-
b. 22q12 Hauser syndrome, except

GYNE TRANS TEAM Page 2 of 16


GYNECOLOGY SAMPLEX
DRAVERGONZ 2022

a. Elevated postpubertal c. Several organs are involves in the


testosterone levels successful function of the
b. Phenotypically female reproductive system
c. Absent uterus and vagina d. Each of the different organs
d. Ovaries are normal involved act independently of
each other
27. One of the ff is NOT indicated for
unicornuate uterus 32. Tanycytes are cells cells in the 3rd ventricle
a. Reconstruction metroplasty thought to be important in the transfer of
b. Cervical cerclage a. GnRH
c. Intravenous pyelogram b. Thyroxin
d. Removal of rudimentary horn with c. Cortisone
functional endometrium d. FSH

28. Effects of androgen exposure on female 33. FSH has the same alpha subunit as the ff,
genitalia before 12 weeks of fetal except:
development a. LH
a. Labioscrotal fusion b. ACTH
b. Retention of urogenital sinus c. TSH
c. Clitoromegaly alone d. hCG
d. Penile urethra
34. FSH is secreted by what organ?
29. Which of the ff statements about congenital a. Ant. Hypothalamus
adrenal hyperplasia (CAH) is incorrect? b. Anterior pituitary
a. Affected fetus has 46, XX genotype c. Post. Hypothalamus
b. CAH presents primarily with d. Ovaries
ambiguous genitalia and/or labial
fusion 35. GnRH-synthesizing neurons originate from
c. CAH is due to deficiency of 17- a. Brain
hydroxyprogesterone b. Pituitary stalk
d. Cortisol replacement is part of c. Neurohypophysis
management of CAH d. Olfactory placode

30. The ff are features of androgen insensitivity 36. Compared with a woman of normal weight,
syndrome, except obese women have increased conversion of
a. Short vaginal pouch a. Estradiol to estriol
b. Normal AMH levels b. Testosterone to progesterone
c. 46, XX karyotype – (46, XY c. Androstenedione to estrone
karyotype) d. Estradiol to androstenedione
d. Sparse or no pubic hair
37. Majority of GnRH neurons controlling the
31. Which of the ff statements is NOT TRUE? HPO axis are located in
a. The reproductive process starts in a. Anterior Hypothalamus
the brain b. Anterior Pituitary
b. The endocrine regulation of the c. Medial Basal Hypothalamus
reproductive process is complex d. Arcuate nucleus

GYNE TRANS TEAM Page 3 of 16


GYNECOLOGY SAMPLEX
DRAVERGONZ 2022

38. Regarding the feedback loop actions of the 44. GnRH is the most active in producing an LH
ovarian steroid hormones on GnRH pulse surge when it is secreted
modulation, which hormone decreases a. Tonically
GnRH pulse amplitude? b. In pulses every 2-4mins
a. Estradiol c. In pulses every hour
b. Progesterone d. In pulses every 3hrs
c. Testosterone
d. Estriol 45. Inhibin, a substance produced in the ovary
a. Is a steroid hormone
39. Which ovarian steroid hormone decreases b. Is synonymous with follistatin
GnRH pulse frequency? c. Inhibits LH more strongly
a. Estradiol d. Stimulates thecal androgen
b. Progesterone production
c. Testosterone
d. Estriol 46. Cervical mucus that can stretch on a slide at
least 6cms is referred to as
40. The structure of the ovum that prevent its SPINNBARKEIT
fertilization by the sperm of another species 47. The ovarian structure evolving from the
is ovulated follicle CORPUS LUTEUM
a. Granulosa 48. The hallmark of the human corpus luteum is
b. Theca interna its secretion of PROGESTERONE
c. Zona pellucida 49. Ovulation occurs about 24 hours after the
d. Vitelline initial rise of LH surge.
50. The second meiotic division of the oocyte
41. There is a direct correlation between the will be completed at time of
increase of serum estrogen during an FERTILIZATION
ovulatory cycle and the 51. The first meiotic division of the oocyte
a. Thickness of the theca externa occurs during FETAL LIFE
b. Size of the dominant follicle 52. The first histologic evidence of ovulation in
c. Thickness of the ovarian cortex the endometrium is the presence of
d. Number of follicles in the ovary SUBNUCLEAR VACOULES in the
endometrial glands
42. Ovulation in the human usually occurs 53. Androgens are converted to estrone or
a. At the same time as the LH peak estradiol by the enzyme AROMATASE
b. Within 24hrs before the LH peak 54. The number of oocytes at birth is 2-4
c. Within 24hrs after the LH peak MILLION
d. At the same time as progesterone 55. The number of oocytes at puberty is
peak 400,000
56. The number of follicles recruited to grow
43. The primary target of FSH activity is the during the follicular phase is 3-7
a. Adenohypophysis 57. The average length of a menstrual cycle is
b. Ovarian granulosa 28 DAYS
c. Ovarian theca 58. The average duration of the menstrual
d. Ovarian hilum phase in a menstrual cycle is 3-5 DAYS.
59. The average blood loss in one menstrual
cycle is 35mL

GYNE TRANS TEAM Page 4 of 16


GYNECOLOGY SAMPLEX
DRAVERGONZ 2022

60. The average age of puberty is 12, and c. Inhibin


menopause is 45-55. (page 93, Compre d. Activin
Gyne 7th Ed.)
8. Which of the following is not an effect of
menopause?
QUIZ 2: MENOPAUSE, PEDIA GYNECOLOGY, a. Hot flushes, sleep disruption, &
BENIGN GYNECOLOGIC LESIONS dementia
b. Vaginal dryness, dyspareunia,
predominance of parabasal cells on pap
1. Menopause is defined by:
smear
a. Age
c. Osteoporosis, bone loss of cortical
b. cessation of reproductive function
bones greater than trabecular bones
c. symptoms (trabecular>cortical)
d. LMP d. Increased risk for coronary artery
disease and myocardial infarction
2. Which of the following determines the age of
menopause? 9. Which of the following is not part of disease
a. Age at coitarche prevention for side effects of menopause?
b. age at menarche a. Bioidentical therapy
c. genes b. Biphosphonates
d. use of OCPs c. screening for disease
d. diet
3. The following may have some effect on onset of
menopause, except: 10. Which of the following is an alternative therapy
a. Age at menarche for symptoms at menopause that does not
b. parity need FDA approval?
c. smoking a. Antidepressants
d. surgery b. Gabapentin
c. Isoflavone
4. The time after cessation of reproductive d. Clonidine
function is referred to as:
a. Menopause 11. Uterine distention due to accumulation of blood:
b. perimenopause a. myoma uteri
c. Climacterium b. hematometra
d. postmenopausal c. endometriosis
d. adenomyoma
5. Premature Ovarian Failure is cessation of
menses before what age? 12. The most common large cyst of the vulva is a
a. 40 y.o. cystic dilatation of this gland.
b. 42 y.o. a. Skene’s gland
c. 45 y.o. b. paraurethral gland
d. 50 y.o. c. Bartholin’s gland
d. cervical glands
6. All of the following hormones are elevated
during menopause, except: 13. The most common benign solid tumor of the
a. Prolactin vulva:
b. FSH a. fibroma
c. LH b. myoloblastoma
d. Activin c. lipoma
d. hemangioma
7. All of the following hormones are decreased
during menopause, except:
a. Estrogen
b. AMH

GYNE TRANS TEAM Page 5 of 16


GYNECOLOGY SAMPLEX
DRAVERGONZ 2022

14. The following are skin infections of the vulva d. cervical polyp
caused by Streptococcus or
Staphylococcus, EXCEPT: 21. Risk factors associated with the development of
a. folliculitis myomas include the following, EXCEPT
b. impetigo a. increasing age
c. furunculitis b. high parity
d. hidradenitis suppurativa c. early menarche
d. tamoxifen use
15. The single most common gynecologic problem:
a. vaginal discharge 22. One of the indications for doing surgery for
b. pelvic pain treatment of myomas are those whose sizes
c. pruritus are equivalent to how many weeks age of
d. none of the above gestation?
a. 8 weeks
16. The most frequent cause of trauma to the lower b. 10 weeks
genital tract of adult females: c. 12 weeks
a. straddle injury d. 14-16 weeks
b. coitus
c. foreign object 23. The presence of ectopic endometrial glands
d. vaginismus and stroma in the uterus:
a. endometriosis
17. Which of the following is NOT TRUE of b. adenomyosis
Nabothian Cysts: c. adenomyoma
a. they are retention cysts of the vaginal d. endometrial polyp
squamous cells (columnar cells)
b. may be multiple in many women 24. True of Adenomyosis, EXCEPT:
c. they are translucent or opaque white or a. secondary dysmenorrhea
yellow b. menorrhagia
d. they are formed by spontaneous healing c. dyspareunia
process d. irregularly shaped uterus
e. no treatment is necessary
25. Cysts of the fallopian tube which are common
18. The following are true of cervical myomas, incidental findings during
EXCEPT: gynecologic operations:
a. most are small & asymptomatic a. Adenomatoid tumors
b. may cause dysuria, urgency & b. hydatid cysts
dyspareunia c. paratubal cysts
c. seldom solitary d. functional cysts
d. majority actually arise from the isthmus
26. A mother brings her 6 year old daughter to the
19. Which of the following is NOT a common cause Emergency Room (ER) for white discharge and
of cervical stenosis of the acquired type? vulvar erythema for the past 2 weeks. The child
a. radiation is hesitant to allow her mother to touch her
b. infection vulvar area. Her mother informs you that
c. neoplasia recently the child is reluctant to go near, or be
d. none of the above touched by, an uncle who lives with the family.
The physical finding most likely to be found in
20. Localized overgrowth of endometrial glands & this child is:
stroma that project beyond the surface of a. Posterior hymenal tear?
the Endometrium? b. Vulvar hemangioma
a. Leiomyoma c. Urethral caruncle
b. Endometrial polyp d. Lichen sclerosus
c. endometriosis

GYNE TRANS TEAM Page 6 of 16


GYNECOLOGY SAMPLEX
DRAVERGONZ 2022

27. You are called to the ER to evaluate a 10 year d. Lichen Sclerosus


old premenarcheal girl with abdominal pain.
Your Physical Examination (PE) is limited by 32. What is the most common cause of
voluntary guarding of the abdomen. An vulvovaginitis?
abdominal ultrasound shows bilateral solid a. Bacterial
adnexal masses each 10 x 10 cm in size. Her b. Protozoal
hCG level is 62 and LDH is 137. What is the c. Nonspecific
most likely diagnosis? d. Mycotic
a. Endodermal sinus tumor
b. Immature teratoma 33. Which of the following is a common cause of
c. Dysgerminoma nonspecific vulvovaginitis?
d. Theca-lutein cyst a. Chemicals (detergents, soaps)
b. Tight-fitting clothing
28. A 17 year old high school student is sent to the c. Poor hygiene and allergens
ER by their school nurse for malaise, d. All of the above
decreased appetite and abdominal pain. Her
history revealed that she has irregular periods. 34. Which of the following is a reason why children
She is sexually active but with a negative hCG. are more susceptible to nonspecific
On PE there is a RLQ pain and rebound vulvovaginitis than adults?
tenderness. What is the next appropriate step? a. Vaginal epithelium has an acidic pH
a. Laparoscopy b. Lack of pubic hair
b. CT Scan of the Abdomen/Pelvis c. Both A and B
c. GI consult d. None of the above
d. Administer antibiotics
35. Labial adhesions does not require treatment
29. In a pediatric patient, for which of the following unless what condition occurs?
conditions would you prescribe clobetasol a. Voiding is compromised
proprionate and inquire about the possibility of b. There is pain
sexual abuse? c. There is a perineal mass
a. Vulvar adhesions d. There is active vaginal bleeding
b. Lichen sclerosus
c. Embryonal rhabdomyosarcoma 36. Which of the following conditions is included in
d. Mullerian agenesis the differential diagnosis of persistent or
recurrent vulvovaginitis?
30. A 7 year old girl presents to the ER with a. Lichen Sclerosus
perineal injury that is actively bleeding. You are b. Urethral Prolapse
told that she fell on the crossbar of her brother’s c. Foreign body
bike. The history obtained from the patient and d. Labial Adhesions
her mother is inconsistent. PE shows no
bruises over the skin and external genitalia but 37. Which of the following organisms is NOT a
with a defect involving the fourchette and cause of specific bacterial prepubertal
extending to perirectal area. Her anal orifice is vulvovaginitis?
intact. What is the next best step? a. Escherichia coli
a. Observation in the ED b. Staphylococcus aureus
b. Laparoscopy c. Group A: β-hemolytic streptococcus
c. Pelvic sonogram d. Trichomonas vaginalis
d. Examination under anesthesia
38. What is the classic symptom of Enterobius
31. What is the most frequent gynecologic disease vermicularis infestation?
of children? a. Itching
a. Vulvovaginitis b. Vaginal discharge
b. Labial Adhesions c. Soreness
c. Urethral Prolapse d. Vaginal bleeding

GYNE TRANS TEAM Page 7 of 16


GYNECOLOGY SAMPLEX
DRAVERGONZ 2022

d. Infertility
39. What is the most common foreign body found in
the vagina of a child? 3. Absolute contraindication for hysteroscopy:
a. Hair pins a. Active bleeding
b. Toilet paper b. Extensive adhesions
c. Crayons c. Leiomyomata
d. Gravel
d. Acute pelvic infections
40. Which of the following is a differential diagnosis
of persistent vaginal bleeding in a 4. A distension media that is antigenic and
preadolescent female? rapidly crystallizes:
a. Neoplasia a. Normal saline
b. Precocious puberty b. Lactated ringer’s sorbitol
c. Urethral prolapse c. Dextran
d. All of the above
5. Equipment for the office hysteroscopy,
41. Two most common congenital causes EXCEPT:
of hematometra: a. Speculum
- Imperforate hymen b. Tenaculum
- Transverse vaginal septum c. Allis
d. Forceps
42. Three most common types of Uterine
Leiomyoma:
- Submucous 6. Appropriate test to request prior to
- Intramural hysteroscopy:
- Subserous a. Pregnancy test
b. Urinalysis
43. Three types of degeneration of myomas: c. Chest x-ray
- Hyaline d. HbsAg
- Cystic
- Red/carneous 7. Best time during the menstrual cycle to
perform hysteroscopy:
44. Two functional benign ovarian cysts: a. Early to middle proliferative phase
- Follicular cyst b. Late proliferative phase
- Corpus Luteum Cyst c. Late secretory phase
d. Early to middle secretory phase
QUIZ 3 : FAMILY PLANNING,
HYSTEROSCOPY/LAPAROSCOPY, GENITAL 8. Most frequent problem in performing
TRACT INFECTIONS hysteroscopy:
FAMANNING a. Cervical stenosis or spasm
b. Bleeding
1. Direct visualization of the endometrial cavity
c. Infection
a. Laparoscopy
d. Endometrial pathology
b. Hysteroscopy
c. Endometrial biopsy
9. Directly visualize pelvic anatomy:
d. Fractional curettage
a. Ultrasound
b. CT scan
2. Hysteroscopy is most frequently used in the
c. Hysteroscopy
evaluation of:
d. Laparoscopy
a. Abnormal uterine bleeding
b. Recurrent miscarriage
10. Advantage of laparoscopy:
c. Uterine synechiae

GYNE TRANS TEAM Page 8 of 16


GYNECOLOGY SAMPLEX
DRAVERGONZ 2022

a. Shorter hospital stay


b. Increased risk of wound infection 18. Besides condom, which is another barrier
c. Thromboembolic events method of birth control?
d. Higher cost a. Diaphragm
b. IUD
11. Most common indication for laparoscopy: c. Withdrawal
a. Female sterilization d. Sterilization
b. Diagnostic
c. Ovarian cystectomy 19. Which type of intrauterine device (IUD) is
d. Hysterectomy available?
a. Copper
12. Relative contraindication for hysterectomy: b. Titanium
a. Morbid obesity c. Hormonal
b. Intestinal obstruction d. A and C
c. Hemodynamic instability
d. Tuberculous peritonitis 20. Which of these is a possible side effect of
birth control pills?
13. Indication for laparoscopy in pregnancy, a. Nausea
EXCEPT: b. Irregular bleeding
a. Appendicitis c. Headaches
b. Ovarian torsion d. All of the above
c. Gallbladder disease
d. Leiomyoma 21. How long is the vaginal ring left in place?
a. 1 week
14. Optimal time to perform laparoscopy b. 2 weeks
surgery in pregnancy: c. 3 weeks
a. Early second trimester d. 3 months
b. Late second trimester
c. Late first trimester 22. Which if these methods is called natural
d. Early third trimester family planning?
a. Tracking basal temperature
15. Major complication of laparoscopy: b. Tracking changes in cervical mucus
a. Laceration of blood vessels c. Tracking the menstrual cycle on a
b. Uterine perforation calendar
c. Fluid overload d. All of the above
d. Anaphylaxis
16. Which methods of birth control needs a 23. Which of these methods of sterilization is
prescription? permanent?
a. Male condom a. Tubal sterilization
b. Female condom b. Vasectomy
c. Cervical cap c. A and B
d. Spermicides d. None of the above

17. What do male condoms offer that other 24. Which of the following is NOT a barrier
forms of birth control do not? contraceptive?
a. Least chance of failure a. Diaphragm
b. Best protection against STIs b. Condom
c. Cheapest to use c. Contraceptive patch
d. All of the above d. Cervical cap

GYNE TRANS TEAM Page 9 of 16


GYNECOLOGY SAMPLEX
DRAVERGONZ 2022

30. Which of the following is NOT a fertility


25. Besides incorrect use, what is another awareness contraceptive method?
reason that condoms may not be effective? a. Cervical mucus method
a. Most men buy the incorrect size b. Body temperature method
b. They do not contain enough spermicide c. Calendar method
c. They can break during intercourse d. Withdrawal method
d. They can break during intercourse
31. A 22 year old woman with severe itchiness
26. Which contraceptive method involves long- in the perineum, wrists and breasts that
acting progesterone injected intramuscularly worsens at night.
every 3 months? a. Allergy
a. Ortho Evra b. Scabies
b. Seasonale c. Molluscum contagiosum
c. Depo-Provera d. Lice
d. Mirena e. Pityriasis rosea

27. Which form of birth control was developed 32. Painless beefy red ulcers on the vulva
in the 19th century and was the first to be a. Syphilis
widely used by women? b. Chancroid
a. The diaphragm c. Herpes simplex
b. The IUD d. Granuloma inguinale
c. Spermicide jelly
d. The pill 33. Bartholin duct infection is usually due to
a. Gonococcus
28. What is meant by the perfect use failure of a b. Chlamydia trachomatis
contraception? c. Polymicrobial flora
a. The number of times in a given year that d. Bacteroides
a woman fails to get pregnant when she e. Escherichia coli
wants to
b. The number of pregnancies that occur 34. Most accurate method for diagnosing acute
while using a particular contraceptive PID
method over an extended period of time a. History
c. The reliability rate of a particular form of b. P.E.
birth control tested in a clinical lab c. Ultrasonography
setting with human volunteers d. Leukocytosis
d. The number of pregnancies (per 100 e. Diagnostic laparoscopy
users)that are likely to occur the first
year of use when a particular 35. Most frequent symptoms of acute PID
contraception method is used a. Vaginal discharge
consistently and correctly b. Abnormal bleeding
c. Nausea & vomiting
29. If a diaphragm is used consistently and d. Lower abdominal pain
correctly, it can be how effective in e. Urinary frequency
preventing pregnancy?
a. Up to 98% 36. A 32 year old multipara with an IUD in place
b. Up to 94% is admitted for acute PID. The IUD should
c. Up to 92% be removed
d. Up to 90% a. As soon as the diagnosis has been
made

GYNE TRANS TEAM Page 10 of 16


GYNECOLOGY SAMPLEX
DRAVERGONZ 2022

b. As soon as antibiotics have been started 42. The most specific test commonly used to
c. As soon as adequate levels of detect human HIV infection
antibiotic has been achieved a. ELISA
d. 24 hours after the antibiotics has been b. Southern blot
initiated c. PCR
e. As the conclusion of parenteral antibiotic d. Northern blot
therapy e. Western blot

37. 62 year old woman is being treated for PID. 43. A 28 year old woman with severe pubic
The differential should include itching and few small brown spots at the
a. Hypothyroidism suprapubic skin has
b. Genital tract malignancy a. Junctional nevi
c. UTI b. Scabies
d. Exogenous hormone exposure c. Yeast
d. Trichomonas infection
38. Which is associated with a reduction in the e. Lice
risk for PID?
a. Vaginal douching 44. The normal vaginal pH
b. IUD use a. 3.8-4.5
c. A history of a tubal ligation b. 4.5-4.0
d. A history of treated pelvic infection c. 4.0-4.8
d. 2.5-3.5
39. The most common organism associated e. 5.0-5.5
with nonpuerperal endometritis
a. Cytomegalovirus 45. A patient asks for the most effective short-
b. N. gonorrhea term treatment of her vulvar warts. The best
c. Chlamydia trachomatis choice is
d. Streptococcus agalactiae a. Electrocautery
(choose your fighter) b. Laser therapy
c. Interferon
40. Clue cells in wet mount d. Excision
a. Chlamydia cervicitis e. Cryotherapy
b. Gonorrhea
c. Yeast vaginitis 46. Matching Type. Set A (diagnostic
d. Trichomonas vaginitis procedure)
e. Bacterial vaginosis 1. Syphilis - RPR
2. PID – Laparoscopy
41. Clue cells are 3. Herpes Simplex - Clinical
a. White blood cell with phagocytised 4. Chlamydia trachomatis - Culture
bacteria 5. Gonococcal infection – PCR
b. Epithelial squamous cell covered
with bacteria 47. Matching Type. Set B (Pathognomonic
c. Epithelial columnar cell covered with feature)
bacteria 1. Sulfur granules - Actinomyces
d. White blood cell containing gram (-) 2. Violin strings on laparoscopy - PID
paired 3. Bubo formation – Lymphogranuloma
e. Squamous cell epithelium containing veneruem
macrophages 4. ‘school of fish’ - Chancroid

GYNE TRANS TEAM Page 11 of 16


GYNECOLOGY SAMPLEX
DRAVERGONZ 2022

5. Groove sign - Lymphogranuloma 5. Danazol treatment of endometriosis is


veneruem associated with an increase in
a. FSH
48. Matching Type. Set C (Treatment) b. LH
1. Chlamydia – Doxycycline c. LDL
2. PID - Cerftriaxone d. HDL
3. Lymphogranuloma Veneruem -
6. The most common surgical approach to
Doxycylcine
endometriosis is
4. Chancre – Penicillin a. Hysteroscopy
5. Herpes Simplex - Acyclovir b. Laparoscopy
c. laparotomy
49. 3 essential clinical criteria to diagnose PID: d. transvaginal excision
(enumerate)
a. Lower abdominal tenderness/ uterine 7. The classic signs of endometriosis include all of
contractions the following except
b. Adnexal tenderness a. a uterosacral ligament nodularity
c. Cervical motion tenderness b. a fixed retroverted uterus
c. bilat. Symmetrical adnexal enlargement
50. 2 minor clinical criteria to diagnose PID: d. tenderness on examination
(enumerate)
8. Endometriosis is associated with all of the
a. Fever following except
b. Vaginal discharge a. anovulation
b. pain
c. first trimester abortion
QUIZ 4 PREOPERATIVE, POSTOPERATIVE
d. dyspareunia
COUNSELLING & MGT., ENDOMETRIOSIS
1. The classic symptom of Endometriosis 9. Which of the following is not a treatment modality
a. cyclic lower abdominal pain for endometriosis?
b. constipation a. NSAID
c. abdominal pain b. Progesterone
d. diarrhea c. GnRH
d. none of the above
2. Endometriosis is defined as the presence of
which of the following in an ectopic site? 10. Endometriosis of the ovary:
a. endometrial glands a. Adenomyoma
b. stroma b. Endometrioma
c. Either c. Endometritis
d. both d. Adenometritis

3. The nature of endometriosis as a disease: 11. Explains endometriosis at rare and remote sites
a. Anatomic a. Metaplasia
b. Inflammatory b. Iatrogenic
c. Immunologic c. Retrograde Menstruation
d. Infectious d. Lymphatic and Vascular Metastasis

4. The most common site for endometriosis: 12. Prevalence of endometriosis in women with
a. rectosigmoid chronic pelvic pain
b. appendix a. 11%
c. ovary b. 33%
d. uterosacral ligament c. 45%
d. 21%

GYNE TRANS TEAM Page 12 of 16


GYNECOLOGY SAMPLEX
DRAVERGONZ 2022

c. The drug of choice is a single drug used for


13. Which of the following does NOT define the overt pelvic infection as the procedure
characteristics of endometriosis? involves the pelvic organs
a. Invasive d. This decreases the growth of gram positive
b. locally infiltrative organisms predominantly found on the skin
c. benign thus diminishing the risk of infection at the
d. non-disseminating time skin incision during surgery is started.

14. The 3 cardinal histologic features of 19. Based on the National Research council
endometriosis, except: Classification of surgical wounds, cesarean
a. Powder burn appearance delivery before the onset of labor is
b. Ectopic endometrial stroma a. Clean wound
c. ectopic endometrial glands b. Clean-contaminated wound
d. hemorrhage into adjacent tissues c. Contaminated
d. Dirty
15. Deep infiltrating endometriosis, except:
a. Medical management 20. Hysterectomy on a 45 year old G3P3 patient
b. Penetrations >5mm due to Myoma uteri is
c. surgical management a. Clean wound
d. retrograde menstruation theory b. Clean-contaminated wound
c. Contaminated
16. Preoperative discussion about the surgery with d. Dirty wound
the patient and her family does not include
a. Making them understand the risk of dying 21. Performing an emergency Cesarean delivery on
from the surgery a patient due to Abruptio Placenta with rupture
b. What the operation will not accomplish of membranes for 30 minutes, and a cervical
c. Making them decide in order to obtain the dilatation of 4 cm.
consent a. Clean wound
d. That the surgery will remove her b. Clean-contaminated wound
anxieties and problems on the organ c. Contaminated
involved?? d. Dirty wound

17. The informed consent 22. Preoperative preparation of patients with


a. Is a written document given by the patient thromboembolic disease who are on anticoagulants
even if she does not understand much includes
about the procedure provided she a. Adjusting the dose to a single dose, long
understands it is for her own good. acting at least a week prior to surgery.
b. Is a verbal or a written document given by b. Shifting the anticoagulant to an oral form, th
the patient in cases of emergency in order given twice daily 5 days before the
to carry out the procedure procedure
c. Is a verbal agreement between the patient c. Stopping the anticoagulant use at least 5
and health worker in order to get a desirable days before the surgical procedure.
result d. Delaying the surgical procedure until the
d. Is given by the patient after she patient will no longer need anticoagulant,
comprehends about the procedure and then schedule her for the surgical procedure
she decides to agrees. 3 days after the last dose of anticoagulant
given
18. Prophylactic antibiotic
a. This prevents postoperative complication 23. There is a higher risk for pelvic infection after
and is given upon induction of anesthesia an elective hysterectomy if the patient has this
before the start of the surgical procedure infection weeks prior to the contemplated
b. Uses broad spectrum antibiotic procedure
especially sensitive to anaerobes a. Candidiasis

GYNE TRANS TEAM Page 13 of 16


GYNECOLOGY SAMPLEX
DRAVERGONZ 2022

b. Hepatitis B infection 28. The most common first sign of morbidity among
c. Bacterial vaginosis elderly patients after a surgical procedure
d. Acute gastroenteritis a. Fever
b. Cough
24. A 48 year old G5P5 is scheduled for elective c. Mental status change
total abdominal hysterectomy because of a Left d. Loose bowel movement
ovarian new growth measuring 10 x 10 cm. The
history revealed on and off abdominal discomfort 29 The goal in the preoperative evaluation of
for 8 months with changes in bowel movements. patients no matter what type of procedure she will
Blood work up and renal function tests were undergo
unremarkable. Stool exam was positive for guaiac a. To reduce the anxieties of the team doing
test. Additional diagnostic tests will include the procedure and prepare them for any
a. Chest X-ray, ECG, Colonoscopy unusualities.
b. Chest x-Ray, 2 D echo, Colonoscopy, CT b. To obtain adequate information about the
scan of the ureters patient and family including the financial
c. Chest x-Ray, ECG, 2 D echo, Colonoscopy, capability and affiliation on health
CT scan of the ureters insurances.
d. Chest x-Ray, ECG, 2 D echo, Colonoscopy c. To minimize surprise for the patient,
health care workers.
25. A 40 year old G2P2 is for hysterectomy due to d. To make sure that the operation is
a cervical myoma. The uterus measures 15 x 10 indicated, considering the patient
cm, with limited mobility. She is a smoker for 10 holistically, not only focusing on the pelvic
years and her BMI is 18. area.
a. She will be advised for practical purposes to
stop smoking at least 2 months before the 30. This does not encompass during the discussion
operation as part of the preoperative of the doctor with her patient before the
preparation contemplated procedure
b. For practical purposes advise her to stop a. The physician may suggest that the patient
smoking at least 1 month before the seek another professional opinion before
contemplated surgery she decides to proceed with the operation
c. She will be advised to stop smoking and b. The patient will ask all kinds of pertinent
return when she is ready for the surgery so questions to the physician including her
the schedule of operation can be definite future capability to conceive
d. Ideally, advise her to focus more on the goal c. The patient discloses to the physician her
to stop smoking but to return anytime if marital status including her sexual
complications set in. relationships, sexual practices to make sure
that her concerns are addressed
26. The risk factor for a surgical procedure begins d. The physician educates the patient about
to increase at this age, thus more diagnostic work- the normal anatomy, what the pathology
ups are necessary. is to assure her that the procedure will
a. 35 relieve all her concerns and anxieties.
b. 40??
c. 50 31. The patient underwent a surgical intervention
d. 55 for 4 hours and the estimated blood loss was 2,000
cc. Postoperatively, the earliest sign that may
27. For surgical procedures done among the indicate decreased intravascular blood volume
elderly, the greatest source of morbidity and a. Tachycardia
mortality are related to the b. Dyspnea
a. Cardiac status c. Low urine output
b. Pulmonary status d. Restlessness
c. Diabetes
d. Renal status 32. The most common diagnostic problem
encountered after a surgical procedure

GYNE TRANS TEAM Page 14 of 16


GYNECOLOGY SAMPLEX
DRAVERGONZ 2022

a. Cough b. Perioperative stabilization


b. Fever c. Return to Normalcy
c. Dyspnea d. Postoperative recovery period
d. Wound infection
39. During this period, the woman is concerned
33. This non pulmonary factor favors atelectasis about how she looks and dealing with the change in
during the postoperative period her body image
a. Incisional pain a. Postoperative recovery
b. Asthma attacks b. Return to Normalcy
c. Retention of secretions c. Flow phase
d. Prolonged immobility d. Perioperative stabilization period

34. The ebb phase 40. The patient after the surgical procedure drifts in
a. Refers to the state of the patient brought and out of an awake state
about by normal physiologic response to a. Return to Normalcy
the stress of the operation b. Perioperative stabilization
b. Results in periorbital edema due to the c. Postoperative recovery
retention of fluid through release of d. Perioperative period
aldosterone
c. The patient experiences brisk diuresis which 41. Recovery from anesthesia is observed during
indicates her postoperative status is this period after a surgical procedure
improving a. Perioperative stabilization
d. This is the state of the patient brought about b. Return to Normalcy
by the surgical procedure as a response to c. Postoperative recovery
the intervention d. Flow phase

35. The first cardiovascular response when the 42. After the surgical procedure, the patient had
patient is in hypovolemic shock intraoperatively and circulatory insufficiency preventing adequate
postoperatively vascular perfusion of vascular organs, you consider
a. Tachycardia a. Septic shock
b. Dyspnea b. Hypovolemic shock
c. Low urine output c. Thyroid storm
d. Restlessness d. Disseminated Intravascular Coagulation

36. During the first 24 to 48 hours after surgery, the 43. A 50 year old G5P5 with BMI 22 underwent
attention of the surgical team is toward the total abdominal hysterectomy with
resumption of her normal physiologic functions salpingoooporectomy due to myoma uteri. Her
a. Ebb phase procedure was technically difficult and lasted for 5
b. Return to normalcy hours. The estimated blood loss was 3000 cc.
c. Postoperative period Postoperative management includes
d. Perioperative stabilization a. Monitor Blood pressure, heart rate,
respiratory rate every 15 minutes
b. Monitor Blood pressure, heart rate,
37 During this period, most traditional complications respiratory rate, urine output every 15
after the surgical intervention becomes apparent minutes
a. Ebb phase c. Monitor Blood pressure, heart rate,
b. Return to normalcy respiratory rate every 15 minutes; urine
c. Postoperative recovery output every 30 minutes
d. Perioperative stabilization d. Monitor Blood pressure, heart rate,
respiratory rate every 15 minutes, and
38. This period encompasses the transmission from urine output every hour
sick role to her usual preoperative status and
activity
a. Flow phase

GYNE TRANS TEAM Page 15 of 16


GYNECOLOGY SAMPLEX
DRAVERGONZ 2022

44. The principle in the fluid replacement for a


woman who had significant blood loss is that for
every 1 ml of blood lost,
a. Replace with 5 ml of crystalloid
b. Replace with 3 ml of crystalloid
c. Replace with 5 ml of colloid
d. Replace with 3 ml of colloid

45. This is considered as the most common cause


for acute circulatory failure in the field of
gynecology
a. Hypovolemic shock
b. Septic shock
c. Anaphylactic shock
d. Thyroid storm

GYNE TRANS TEAM Page 16 of 16


GYNECOLOGY SAMPLEX THIRD BIMONTHLY
S.Y. 2020-2021 | DRAVERGONZ 2022

1. FSH has the same alpha subunit as the C. FSH


following hormone, EXCEPT D. Progesterone
A. LH
B. TSH 10. Which of the following is an abnormal menstrual
C. hCG cycle/menstrual period?
D. oxytocin A. 35 days long
B. 80 ml vol.
2. GNRH synthesis C. menstrual phase of 4 days
A. Hypothalamus D. spotting for 24 hrs
B. Ant. Pituitary
C. Post. Pituitary 11. A 26-year old woman complains of vaginal
D. Ovary discharge with itching & burning. pH of the
discharge is 5.5. Which of the following is the
3. FSH synthesis most likely diagnosis?
A. Hypothalamus A. Chlamydia cervicitis
B. Ant. Pituitary B. Yeast inf.
C. Post. Pituitary C. Trichomonas vaginitis
D. Ovary D. bacterial vaginosis

4. LH effect: 12. Gumma, an infectious granuloma is


A. Hypothalamus characteristic of what stage of syphilis?
B. Ant. Pituitary A. primary stage
C. Post. Pituitary B. secondary stage
D. Ovary C. late stage
D. neurosyphilis
5. Estrogen synthesis:
A. Hypothalamus 13. The pathognomonic clusters of dark-staining
B. Ant. Pituitary bacteria (bipolar in appearance) found in the
C. Post. Pituitary Cytoplasm of large mononuclear cells in patients
D. Ovary with Granuloma inguinale?
A. Clue cells
6. In a 30-day menstrual cycle, when is peak of LH B. Donovan bodies
surge expected? C. Arias-stella reaction
A. Day 13 D. Hobnail pattern
B. day 14
C. day 15 14. Transvaginal sonographic finding of acute
D. day 16 salpingitis render?
A. violin string sign
7. In a woman’s reproductive life, there will be an B. atoll sign
average of how many opportunities to become C. Cogwheel appearance
Pregnant? D. Scimitar sign
A. 4000,000
B. 2 million 15. A 21-year old visited her OB-GYNE complaining
C. 400 of abdominal pain, irregular bleeding a few
D. 40 weeks ago and foul-smelling vaginal discharge
beforehand. Recently she noted right upper
8. This is secreted by the corpus luteum. quadrant pain, pleuritic, with tenderness in the
A. Estrogen right upper quadrant. The pain radiated to the
B. progesterone shoulder and back. The most likely differential
C. LH diagnosis would be:
D. FSH A. pneumonia
B. Acute cholecystitis
9. This hormone supports the corpus luteum. C. Fitz-Hugh-Curtis syndrome
A. hCG D. Mittelschmerz
B. LH

GYNE Trans Team Page 1 of 9


GYNECOLOGY SAMPLEX THIRD BIMONTHLY
S.Y. 2020-2021 | DRAVERGONZ 2022

16. “Silent PID” C. PCR


A. Chlamydia D. Western blot
B. Gonorrhea
C. Tuberculosis 23. A 23-year old woman complains of severe pubic
D. Bacteroides itching. On examination, a few small brown
spots are seen on the suprapubic skin. The most
17. Which of the following women is likely to have a likely diagnosis is
Bartholin gland carcinoma? A. junctional nevi
A. 13-year old pubertal girl B. scabies
B. 22-year old sexually active woman C. yeast
C. 39-year old monogamous married D. lice
woman
D. 52-year old virginal woman 24. In a tuboovarian complex associated with acute
PID, the flora is predominantly
18. Which of the following is NOT a management for A. Group D Enterococcus
Bartholin’s Abscess? B. mixed Anaerobes
A. topical ointments C. Chlamydia trachomatis
B. marsupialization D. Neisseria gonorrhea
C. I&D
D. Antibiotics 25. The most frequent symptom of acute PID is
A. vaginal discharge
19. A 16-year old examined during her menstrual B. abnormal bleeding
period has hypertension, skin rash, fever, & C. nausea & vomiting
myalgia. She is found to have Staphylococcus D. lower abdominal pain
on her vaginal culture. The girl’s mother asks
you to explain. Her daughter’s illness & asks 26. The average age of menopause for Filipinas
about proposed treatment plan. You should tell A. 45-46 y.o
her that B. 47-48 y.o
A. the symptoms are due to C. 49-50 y.o.
Staphylococcus aureus bacteremia D. 51 y.o.
B. Almost all cases are sexually
transmitted 27. Which of the following is not one of the causes
C. Antibiotic treatment is of no help of Premature Ovarian Failure?
D. She should stop using tampons A. Increased rate of germ cell use
B. Decreased germ cell endowment
20. Long term sequelae of PID does NOT include C. increased germ cell destruction
which of the following? D. Desarda hernia repair
A. ectopic pregnancy
B. chronic pelvic pain 28. All are etiologies of Premature ovarian Failure,
C. recurrence except:
D. infertility A. Gonadal dysgenesis
B. Galactosemia
21. A 22-year old woman complains of severe C. either of A & B
itching in her perineum, wrists& breasts which D. none of the above
worsen at night. Examination reveals
excoriations on these areas. The most likely 29. In managing Premature ovarian failure, the
diagnosis is: following are done except:
A. allergy A. Transvaginal ultrasound
B. scabies B. Estrogen replacement
C. Molluscum contagiusum C. oocyte donation
D. Lice D. none of the above

22. The most specific test to detect HIV infection is 30. Which is NOT an ovarian change at
A. ELISA perimenopause?
B. Southern blot

GYNE Trans Team Page 2 of 9


GYNECOLOGY SAMPLEX THIRD BIMONTHLY
S.Y. 2020-2021 | DRAVERGONZ 2022

A. Marked diminution of reproductive B. Immunologic


capacity C. Retrograde Menstruation
B. Low antral follicle count D. Hematogenous Spread
C. rising FSH
D. rising estrogen 38. Management of Deep Infiltrating Endometriosis:
A. NSAIDs
31. Treatment at perimenopause is addressed to all B. SERM
of the following, except: C. Surgery
A. abnormal uterine bleeding D. Danazol
B. vaginal dryness
C. hot flushes 39. Etiology of ovarian endometriosis:
D. inability to conceive A. Coelomic Metaplasia
B. Immunologic
32. The most reliable method to establish bone C. Retrograde Menstruation
mineral density is: D. Hematogenous Spread
A. Computed Tomography (CT)
B. Dual-photon absorptiometry 40. Features of Mayer-Rokitansky-Kuster-Hauser
C. single photon absorptiometry syndrome, EXCEPT
D. dual-energy x-ray absorptiometry A. elevated postpubertal testosterone
(DEXA) levels
B. phenotypically female
33. A patient came for check up because of a mass C. absent uterus and vagina
at the vulva which made walking difficult. Upon D. ovaries are normal
inspection, you noticed that it was at around 5
o’clock position and is tender. What is your 41. Hormonal milieu seen in Congenital adrenal
diagnosis? hyperplasia
A. Herpes Simplex infection A. decreased dehydroepiandrosterone
B. Bartholin’s abscess B. normal cortisol level
C. inflamed skene’s duct C. elevated androstenedione
D. PID D. increased aldosterone

34. The patient wants immediate relief of pain and 42. DES-related mullerian anomaly
needs to walk comfortably. What will be the A. “banana-shaped uterus”
most appropriate management? B. T-shaped uterus
A. Incision and drainage C. labioscrotal fusion
B. Hot sitz D. transverse vaginal septum
C. antibiotics
D. marsupialization 43. Effects of androgen exposure on female
genitalia before 12 weeks of fetal development,
35. The most common site for ectopic endometrial EXCEPT
glands and tissues outside of the endometrium? A. labioscrotal fusion
A. Ovary B. retention of urogenital sinus
B. previous incision site C. clitoromegaly alone
C. peritoneum D. penile urethra
D. nose
44. One of the following is NOT indicated for
36. Which of the following does not refer to unicornuate uterus
endometriosis? A. reconstruction metroplasty
A. Abnormal uterine bleeding B. cervical cerclage
B. Nodularities at the uterosacral area C. intravenous pyelogram
C. cyclic hypogastic pain D. removal of rudimentary horn with
D. infertility functional endometrium

37. Etiology of Deep Infiltrating Endometriosis: 45. How is septate uterus best diagnosed?
A. Coelomic Metaplasia A. hysteroscopy

GYNE Trans Team Page 3 of 9


GYNECOLOGY SAMPLEX THIRD BIMONTHLY
S.Y. 2020-2021 | DRAVERGONZ 2022

B. hysteroscopy and laparoscopy A. 15


C. saline infusion sonography B. 35
D. laparoscopy C. 55
D. 75
46. Imperforate hymen
A. membrane is located at the lower third 51. Which of the following is the most commonly
of the vagina a distance from the reported form of incest?
introitus A. Brother-sister
B. manifests even before puberty B. Father-daughter
C. due to defect in fusion of urogenital C. Father-son
sinus and sinovaginal bulbs D. Mother-son
D. can predispose patient to
endometriosis 52. A father brings his 9-year-old daughter to the
office after he picked her up for joint custody
47. Pathogenesis of Androgen insensitivity visit. He and his ex-wife have been in a long
syndrome drawn-out custody battle. His daughter told him
that her mom’s new boyfriend was touching and
A. defective Leydig cells that produce
poking her “down there” last night while mom
testosterone
was shopping.
B. absence of testes When childhood sexual assault is suspected
C. presence of defective androgen within the past 72 hours, which of the following
receptors should be the next action of the physician?
D. due to defective target tissues of A. Bring family members together for an
androgens interview
B. Contact mental health workers
48. A mother brings her 4-year-old daughter in for C. Notify the police
complaints of itching “down there” and staining D. Perform a complete physical exam
on the underwear. Which of the following
condition is the most likely cause of vulvovaginal 53. Which of the following is a legal but not a
symptoms in children? medical responsibility of the physician caring for
an alleged sexual assault victim?
A. Foreign body
A. Collecting samples of hair and
B. Lichen sclerosis vaginal secretions, and microscopic
C. Nonspecific evaluation of motile sperm
D. Physiologic leukorrhea B. Obtaining a complete gynecologic
history
49. A 6-year-old girl is referred by her pediatrician C. Obtaining informed consent from patient
for a friable mass in the genital region. You D. Offering postcoital hormonal prophylaxis
suspect a urethral prolapse. Which of the to prevent pregnancy if reproductive age
following is the most common symptom of
urethral prolapse in the prepubertal, 54. An 8-year old girl is brought to your office soon
unestrogenized girl? after suffering a fall on her brother’s bicycle. Her
A. Dysuria mother reports that the girl’s foot slipped off the
bicycle pedal, which resulted in the girl falling on
B. Hematuria
the center bar of the bicycle. The girl complains
C. Painless genital bleeding
of sharp pain between her legs. There has been
D. Urinary frequency no obvious bleeding and no other injuries are
apparent. The girl is in moderate distress with a
50. An 8-year-old girl is brought in by the mother pulse of 110 bpm, blood pressure of 118/68
after finding her crying and having bloody mmHg, and respiration of 28/min. Physical
underwear. She will not tell her mother what examination is normal with the exception of
happened. On examination, there are injuries inspection of the vulva where a 6-cm tender
consistent with vaginal penetration. You advise bluish mass is present in the area of the right
the mother that it is very important to allow the labia majora. No further examination is possible
authorities to speak with the daughter about because of the girl’s discomfort. Which
what happened. What is the percentage of management is most directly related to an
sexually abused children who know their uncomplicated outcome?
A. Topical application of ice
assailant?

GYNE Trans Team Page 4 of 9


GYNECOLOGY SAMPLEX THIRD BIMONTHLY
S.Y. 2020-2021 | DRAVERGONZ 2022

B. Use of prophylactic antibiotics mercury with varying success and complication.


C. Bed rest for the next 24 hours Currently available spermicides destroy
D. Examination under anesthesia spermatozoa primarily by which of the following
methods?
55. Which of the following is the most direct public A. Activating acrosomal enzymes
health or socioeconomic effect of contraceptive B. Disrupting cell membranes
use? C. Inhibiting glucose transport
A. Improved socioeconomic status D. Altering vaginal enzymes
B. Stabilized world population growth
C. Reduced maternal morbidity 60. A 19-year-old woman and her boyfriend wish to
D. Diminished incidence of fetal use condoms as a barrier contraceptive method.
abnormalities This couple should be advised that the most
reason for failure (pregnancy or STD
56. A premedical student presents requesting transmission) is which of the following?
reversible contraception. She is healthy without A. Breakage
any problems and a normal examination. As you B. Inconsistent use
review her options, she asks which method is C. Leakage caused by spermicidal creams
most reliable. Which of the following D. Use without concomitant use of a
contraceptive methods has the lowest spermicide
pregnancy rate in 100 women using the method
perfectly for 1 year (100 woman-years of use)? 61. An 18-year-old woman presents for care
A. Copper-containing intrauterine because a condom broke during sexual
contraceptive device (IUCD) intercourse. Coitus occured 1 day ago when she
B. Long-acting progestins (Depo-Provera) was at midcycle. She does not wish to be
C. Diaphragm pregnant and will terminate the pregnancy if
D. Oral contraceptives menses does not occur. Regarding her fear of
pregnancy, which is the most appropriate next
57. A 23-year-old woman and her husband wish to step in her management?
use natural family planning as their A. Advise her that unprotected midcycle
contraceptive method. Her menstrual cycle cootus has a 5% risk of pregnancy
length is variable, ranging from 26 to 32 days. B. Prescribe intravaginal misoprostol
She does not plan to measure her basal body (Cytotec)
temperature (BBT). C. Advise immediate douching
The time of her fertility, with the first day of D. Prescribe a brief course of
menses defined as day 1, would be between levonorgestrel
which cycle days?
A. 1 and 14 62. A 42-year-old patient (G3P3(3003)) requests a
B. 6 and 14 diaphragm for contraception. When fitting the
C. 6 and 21 contraceptive diaphragm, it should sit
D. 14 and 21 comfortably between which of the following?
A. Anterior and posterior vaginal fornices
58. You mention that by adding BBT curve, they B. Anterior vaginal fornix and posterior
may be able to determine more effectively when urethrovesical angle
ovulation has occured. The figure below shows C. Pubic symphysis and anterior vaginal
the basal temperature graph made by the couple fornix
the previous month. Which letter most closely D. Pubic symphysis and posterior
identifies when ovulation may have occured? vaginal fornix
A. Point A
B. Point B 63. Which among the following are contraindication
C. Point C to hysteroscopic procedures?
D. Point D A. Cervical intraepithelial neoplasia I
B. Tuberculous Endometritis
59. One of the initial attempts at contraception C. Mitral valve prolapse
involved placement of various items in the D. Uterine cavity depth at 13
vagina to prevent sperm from reaching the
uterus. Historically, these have included
crocodile dung, honey, and preparations with

GYNE Trans Team Page 5 of 9


GYNECOLOGY SAMPLEX THIRD BIMONTHLY
S.Y. 2020-2021 | DRAVERGONZ 2022

64. The most common step in a hysteroscopic A. Discussion occurrence of shoulder pain
procedure where uterine perforation may occur post op is unnecessary since it is
during the: frequent
A. Dilatation of the cervix B. Routine consent for exploratory is not
B. Visualization of cornual regions recommended since the procedure is
C. Fundal visualization in anteflexed simple
uterus C. The patient may choose expectant
D. Direct vaginoscopy while inserting scope management even if endometriosis
associated infertility is suspected
65. True of normal saline as distention medium D. Separate consent for chromotubation
A. Inferior to carbon dioxide in terms of not required since it is part and parcel of
visualization during diagnostic diagnostic laparoscopy
procedure.
B. Suitable for diagnostic hysteroscopy 70. An informed consent by the patient
where monopolar electrosurgery is A. Agreeing that she will abide with the
used. procedure such as taking of vital signs,
C. Not recommended for bipolar operative insertion of catheters and canula
hysteroscopy where glycine is preferred. B. Indicates that the patient understood
D. Results in lesser intravasation about the procedure, what she will go
compared with low viscosity media. through, including the risks, willingly
and voluntarily abides to it
66. Agents which can be utilized to aid in cervical C. Indicates that the patient and her family
dilatation include: members understood the situation, what
A. Oxytocin drip she will go through, and the financial
B. Topical bupivacaine responsibility to compensate the
C. Glycine polymer surgical team and institution for the
D. Vasopressin service rendered
D. Depicts that the patient and her family
67. Appropriate practice in diagnostic hysteroscopy were educated about her condition, the
indication for surgery, and she decides
include:
to proceed with the surgery despite her
A. Routine antibiotic prophylaxis
hesitation since she is aware that her
B. NSAID analgesia before and/or after the
partner’s family is fully financing her
procedure
needs, so she takes the opportunity in
C. Trendelenburg position during
order to avoid being blamed for
procedure
complications that would set in if she
D. Paracervical block preferred over
refuses to undergo the procedure.
topical lidocaine
71. A person who undergoes a surgical procedure is
68. A 27-year old post IVF primigravid with an
at risk for thrombosis, thrombophlebitis,
amenorrhea of 6 weeks was referred for
embolism due to several factors, but the
abdominal guarding and a positive pregnancy
cornerstone that facilitates its development is
test. Ultrasound reveals an extrauterine
the
gestational sac. Which situation would make
A. Activation of the extrinsic and intrinsic
emergency laparotomy the preferred procedure
system of coagulation brought about by
compared with laparoscopy? the procedure
A. A surgeon trained in diagnostic B. Prolonged immobility during the
laparoscopy operation
B. Blood pressure of 100/70 C. Injury to the blood vessels during the
C. No fluid in cul-de-sac on transvaginal surgical procedures brought about by
ultrasound clamping and splitting of tissues
D. Ampullary location of gestational sac in D. Necrosis of the injured tissues thus
oviduct reducing the oxidation-reduction
potential of tissues
69. An anxious patient wishes to discuss diagnostic
laparoscopy with you. She is suspected of 72. History of recurrent preterm birth from
having endometriosis and is undergoing fertility incompetent cervix
work-up. Which among the following is
appropriate advice to the patient?

GYNE Trans Team Page 6 of 9


GYNECOLOGY SAMPLEX THIRD BIMONTHLY
S.Y. 2020-2021 | DRAVERGONZ 2022

A. History of recurrent preterm birth D. Wound infection


from incompetent cervix
B. History of intrauterine fetal death during 78. A 35 year old G0 underwent surgery for an
labor and delivery from asphyxia intrauterine myoma with the size equivalent to a
C. History of heart attack at 35 years old 7 month pregnancy. The estimated blood loss
D. History of stroke among first degree was 3,000 cc. Postoperatively, her vitals signs
relatives especially noted during were stable. After 2 hours, she became
postmenopausal period or after hypotensive. The immediate management will
retirement
be
A. Give oxygen to establish cellular
73. A Pulmonary function test is performed
perfusion
preoperatively on a patient who will undergo
B. Insert another IV line and replace the
surgery in the presence of the condition blood loss with colloids, crystalloid and
A. Smoker for the past 5 years blood
B. BMI 29 C. Secure blood and transfuse, check on
C. History of chronic obstructive the source of hypovolemia
pulmonary disease D. Establish IV line, secure blood, give
D. Surgical procedure will be for a duration oxygen, insert a catheter to measure
of at least 4 hours urine output.
74. When contemplating a surgical procedure in the 79. Symptoms of postoperative wound
elderly due to a large ovarian new growth, the complications/infection first appear
following should be considered A. On the 2nd day after surgery
A. Bowel preparation or cleansing is done B. On the 3rd day after surgery
when the patient is admitted with an IV C. On the 5th after surgery
fluid given. D. On the 14th day after surgery
B. Bowel preparation is done at the
minimal number of sessions in the 80. A 45 year old G5P5 with BMI 18 underwent an
hospital to lessen her agitation and non- abdominal surgery through a midline vertical
submission to the procedure infraumbilical incision. The procedure lasted for
C. Bowel preparation and cleansing may
3 hours. She was discharged apparently well on
be started home before she is
the 3rd post-operative day with instructions on
admitted using oral meds, then once
upon admission a day before surgery home medications. After 1 week she returned on
D. Bowel preparation is not routine a wheelchair, complaining of difficulty in walking.
considering the frailty of tissues and the You consider
physical signs of aging. A. Injury to the ilioinguinal nerve through
the midline vertical incision.
75. Wound dehiscence usually does not involve the B. The iliohypogastric nerve was trapped
separation of the and sutured during the closing of the
A. Peritoneum abdominal incision.
B. Fascia C. The femoral nerve was injured due to
C. Muscles the prolonged and forceful traction
D. Subcutaneous tissue using the abdominal retractor during
the procedure.
76. Abdominal hysterectomy operative procedure D. The femoral nerve was devascularized
based on the National Research Council and denervated when the midline
vertical incision was done.
Classification of Surgical Wounds is classified as
A. Clean
81. The flow phase
B. Clean-contaminated
A. This is the state of the patient brought
C. Contaminated
about by normal physiologic response to
D. Dirty
the stress of the operation
B. Results in periorbital edema due to the
77. Post-operatively, the most common cause of
retention of fluid through release of
temperature elevation is
aldosterone.
A. Presence of retained foreign body C. The patient experiences brisk
B. Atelectasis diuresis which indicates her
C. Pneumonia postoperative status is improving

GYNE Trans Team Page 7 of 9


GYNECOLOGY SAMPLEX THIRD BIMONTHLY
S.Y. 2020-2021 | DRAVERGONZ 2022

D. This is the state of the patient brought 88. Implantation involves the following process
about by the surgical procedure as a except?
response to the intervention A. Apposition
B. Invasion
82. This is considered the foundation of any C. Both
approach to the management of wound D. None of the above
infections
A. Use of 3rd generation antibiotics 89. How many days after ovulation during the
before the start of surgical procedure implantation stage the lacunar network forms?
B. Encouraging to stop smoking and A. 8-9 days
weight loss before the contemplated B. 10-11 days
procedure C. 11-12 days
C. Maintenance of the Hemoglobin and D. 9-10 days
Hematocrit level to the optimum
D. Prevention by considering local and 90. When does the primitive streak starts to form at
systemic factors of the patient that will the caudal end of the embryonic disk?
affect surgery A. Third week after fertilization
B. Fifth week after fertilization
83. Bilateral Tubal Ligation based on the National C. Second week after fertilization
Research Council Classification of Surgical D. None of the above
Wounds is classified as
A. Clean 91. The mendelian inheritance pattern includes the
B. Clean-contaminated following
C. Contaminated A. Aneuploidy
D. Dirty B. X linked trait
C. Both
84. Surgical removal of the ovary based on the D. None of the above
National Research Council Classification of
Surgical Wounds is classified as 92. Type of chromosomal abnormality wherein two
A. Clean or more genetically different cell lines is present
B. Clean-contaminated in the same individual?
C. Contaminated A. Translocation
D. Dirty B. Robertsonian pattern
C. Mosaicism
85. Which of the following refers to the observable D. None of the above
characteristics of an individual?
A. Genome 93. The most significant characteristics of a
B. Genotype mitochondrial disease due to mutation of the
C. Phenotype mtDNA is?
D. None of the above A. Abnormal number of chromosomes
B. Mendelian inheritance
86. What is the substage of prophase where all the C. Maternal inheritance
homologues have paired already and forms a D. Paternal inheritance
tetrad?
A. Zygotene 94. The following is the karyotype of Klinefelter
B. Leptotene Syndrome?
C. Pachytene A. 47, XXX
D. Diplotene B. 47, YXX
C. 47, XYY
87. At what stage of cell division is the egg when D. None of the above
released from the ovary in most mammals
including humans? 95. The following are examples of microdeletion
A. Metaphase II stage abnormality of chromosomes except?
B. Meiosis I stage A. DiGeorge syndrome]
C. Mitosis stage B. Cat’s eye syndrome
D. None of the above C. Neurofibromatosis
D. None of the above

GYNE Trans Team Page 8 of 9


GYNECOLOGY SAMPLEX THIRD BIMONTHLY
S.Y. 2020-2021 | DRAVERGONZ 2022

96. At what period of life does teratogenic events


occur causing cardiovascular malformations?
A. Embryogenic period
B. Fetal period
C. Implantation period
D. Fertilization period

97. Of the 3 billion base pairs of the whole human


genome, how many percent of the assembled
sequences codes for protein?
A. 2%
B. 1%
C. 1.5%
D. 5%

98. What type of genomic variation does not involve


a change in nucleotide sequence?
A. Genetic variation
B. Epigenetic variation
C. Both
D. None of the above

99. What type of abnormality involves only a change


of a single base?
A. Deletion
B. Translocation
C. Duplication
D. Point mutation

100. Bonus

GYNE Trans Team Page 9 of 9


GYNECOLOGY SAMPLEX
4th Bimonthly

QUIZ 6: ABNORMAL UTERINE BLEEDING


(AUB), PREMENSTRUAL SYNDROME (PMS), b. When the volume is around 8 ml per day
AND PREMENSTRUAL DYSPHORIC lasting for 3 days, occurring every 28 days
DISORDER (PMDD) c. when the menstrual flow is around 10 ml
per cycle, lasting for 3 days occurring
1. Chronic Abnormal uterine bleeding every 40 day
a. The condition has been occurring regularly d. when the menstrual flow is 80 ml per cycle,
for the past months lasting for 4 days, occurring every 35 days
b. Prolonged, bleeding occurring 2 x a month
for 3 months 6. To determine the regularity of the menstrual
c. Bleeding has caused the woman to be cycle,
clinically unstable a. The cycle to cycle variation was observed
d. The condition existed for the past 6 over the past 8 months
months and the patient is clinically b. The cycle to cycle variation has to be
stable observed over the past 2 consecutive
months
2. The first line of treatment for a woman with c. The cycle to cycle variation was noted over
Heavy menstrual bleeding the 2 succeeding months
a. Dilatations and curettage d. The cycle to cycle variation was noted
b. Transvaginal ultrasonography over the past 12 months
c. Medical management
d. Endometrial biopsy 7. A 15 year old sought consult about her menstrual
cycle. Her menarche was at 11 years old. She asks
3. A structural category for a woman with AUB about the normal menstrual cycle. You advise her
a. A 35year old woman with heavy menstrual that
bleeding diagnosed with dengue a. The duration of a normal menstrual flow is
hemorrhagic fever at least 3 days
b. A 15year old with Idiopathic b. The minimum number of days for normal
Thrombocytopenic purpura presenting with menstruation is 4 days
prolonged menses c. The duration of a normal menstrual period is
c. A 52 year old woman with up to 8 days
postmenopausal bleeding due to d. The duration of menstruation is determined
endometrial carcinoma by her age of onset of menarche and the
d. A 45year old woman with breast cancer age of menopause of her mother
taking Tamoxifen had vaginal bleeding for 2
months 8. Abnormal uterine bleeding
a. Mostly occurs among the teenagers
4. A 15year old student sought consult at the clinic b. Mostly occurs among the perimenopausal
because she noticed that her menstrual flow is light age
compared to the previous months. She looks pale c. In most cases, the period is shortened
but her vital signs are normal. d. Occurs during the woman’s reproductive
a. This indicates that she has a missed period age and is usually an anovulatory
and is pregnant bleeding
b. The volume of her menstrual blood is only
10 ml for the whole cycle 9. Occurs during the woman’s reproductive age and
c. The volume of her menstrual blood is is usually an anovulatory bleeding
only 3 ml for the whole cycle a. has more estrogen which stimulates
d. This shows an abnormal uterine bleeding secretion of Prostaglandin F2α
and needs to have a transvaginal b. has more progesterone necessary to
ultrasound increase the secretion of prostaglandin
F2α
5. The menstrual cycle is considered infrequent c. has more estrogen which stimulates
a. when it occurs every 21 days secretion of Prostaglandin E2

GYNE TRANS TEAM Page 1 of 10


GYNECOLOGY SAMPLEX
4th Bimonthly

d. has less estrogen necessary to increase the 15. A 30 year old G5P5 has abnormal uterine
secretion of Prostaglandin F2α bleeding for the past 3 months. At present, she is
on her 15 day of menses. Her ultrasound showed
10. When there is anovulation an endometrial lining that is thickened. She was
a. The PGF2ά /PGE₂ ratio steadily increasing prescribed with progesterone as her medication.
from midcycle to menses a. Progesterone will cause endometrial
b. The PGF2ά /PGE₂ ratio remains the same proliferation and this will stop the bleeding
from midcycle to menses b. Progesterone will cause endometrial
c. The PGF2ά /PGE₂ ratio is not altered from proliferation followed by regular shedding of
midcycle to mense the endometrium
d. The PGF2ά /PGE₂ ratio is reduced from c. Progesterone will cause organized growth
midcycle to menses of the endometrium and will result in uniform
sloughing off the endometrium
11. A 35 year old G3P3 has heavy menstrual d. Progesterone will cause organized
bleeding for the past 3 months because of proliferation of the endometrium until spiral
Leiomyoma. The myoma that causes her abnormal arteries undergo vasoconstriction causing
uterine bleeding is mostly likely cessation of menses
a. Intramural
b. Submucous 16. A 25 year old is having headache, depression
c. Subserous and irritability at work for the past week. On that
d. Cervical type day the Head form the Human Resource office
conducted a clinical/Written exam on the
12. A 30- year old G4P4 came in because of candidates for promotion, which included her. She
abnormal uterine bleeding for the past 2 months. topped the exam despite her condition. This
Her last vaginal delivery was 4 months ago and statement will apply in relation to her premenstrual
since she desired pregnancy spacing, an status.
intrauterine device was inserted immediately a. She has PMS since her work is not affected
postpartum. Based on the FIGO classification, she by her behavior
has b. She has PMS since her cognition is not
a. AUB-C affected by her signs and symptoms
b. AUB – I c. She has PMS since her behavior did not
c. AUB-N affect her work
d. AUB- O d. She has PMDD because her behavior has
been there for the whole week
13. A 28 year old G0, single with one sexual
partner had an abnormal uterine bleeding. She was 17. Serotonin level is secreted in large amounts
diagnosed to have Hypothyroidism for the past year a. Brain
and on medication, but she had poor compliance. b. Platelet
All other tests were normal. Based on FIGO c. Blood
classification, she is classified to have d. Gastrointestinal tract
a. AUB-C
b. AUB- I 18. Serotonin is found least in the
c. AUB-N a. Brain
d. AUB-O b. Platelet
c. Blood
14. The first line diagnostic tool to assess structural d. Gastrointestinal tract
abnormalities in a 40 year old woman with
abnormal uterine bleeding 19. This is not a common somatic finding among
a. Pregnancy test patients with premenstrual symptom
b. Transvaginal ultrasound a. Emotional outburst
c. Coagulation test b. Thirst
d. Hematologic test c. Headache
d. Change in bowel habits

GYNE TRANS TEAM Page 2 of 10


GYNECOLOGY SAMPLEX
4th Bimonthly

20. The most common complaint seen in patients 26. Laboratory work up to aid in the Diagnosis of
with premenstrual syndrome and premenstrual PMS and PMDD
dysphoric disorder a. Electroencephalography
a. Irritability b. Calendar Dating
b. Crying c. Saline Infusion Sonography
c. Loss of concentration d. No applicable test
d. Depression
27. Recommended in the regular management of
21. The basis for Diagnosis of Premenstrual PMS
Dysphoric Disorder
a. The presence of a headache, breast a. Calcium supplementation 300 mg daily
tenderness, acne, hot flushes affecting b. Selective Serotonin Reuptake Inhibitor
interpersonal relationship during the 2nd c. Diuretic
half of menstrual cycle d. Low fat diet
b. The presence of PMS symptoms and
persistent irritability causing interpersonal 28. Recommended in the management of PMDD
conflict on the 1st day of menstrual cycle
c. The presence of PMS symptoms and a. Calcium supplementation 300 mg daily
persistent anger causing interpersonal b. Selective Serotonin Reuptake Inhibitor
conflict on 2nd half of the menstrual c. Diuretic
cycle d. Low fat diet
d. The presence of a headache, breast
tenderness, depression, acne affecting 29. A 35 year old with Premenstrual syndrome is
interpersonal relationship during the 1st half given Serotonin Selective Reuptake inhibitor as
of menstrual cycle treatment for her condition. The potential side
effects may include
22. According to the DSM Criteria for PMDD, the a. Progression of breast tenderness
affective symptoms do not include b. Alteration in sleep
a. Feeling of hopelessness c. Perceived body swelling
b. Mood swings d. Persistent thirst
c. Persistent anger
d. Anxiety 30. Oral contraceptives as management for PMS
e. Headache will provide relief of the following symptom
a. Abdominal cramping
23. The most common physical symptom with PMS b. Breast tenderness
a. Change in bowel habit c. Irritability
b. Headache d. Fatigue
c. Breast tenderness
d. Dizziness
QUIZ 7: IEN OF THE LOWER GENITAL
24. This had not been proven to be a cause for TRACT; NEOPLASTIC DISEASES OF CERVIX,
PMS VULVA, AND VAGINA; NEOPLASTIC
a. Poor dietary habit DISEASES OF UTERUS OVARY AND
b. Hypoglycemia FALLOPIAN TUBE; GTD/GTN
c. Prolactin excess
d. Hormonal imbalance 1. CIN
25. A risk factor for PMS A pap smear result showed polygonal-shaped
a. Menarche at 15 years old squamous cells with small nucleus detail can be
b. History of PMS of partner’s mother seen. What is the next step in the management?
c. No sexual partner a. Annual Conventional Pap Smear?
d. BMI of 20 b. Repeat Pap Smear after 6 months
c. HPV Testing
d. Colposcopy

GYNE TRANS TEAM Page 3 of 10


GYNECOLOGY SAMPLEX
4th Bimonthly

a. Type 1 or extrafascial hysterectomy with


2. A woman came in for cervical cancer bilateral salpingo-oophorectomy is the
screening, which of the following would you treatment of choice
advise? b. Whole abdomen CT Scan is requested to
a. Start screening 3 years after sexual evaluate the pelvic and para-aortic lymph
debut but not less than 28 y/o nodes
b. Conventional pap smear is done c. Pelvic external beam RT without
annually brachytherapy is applicable for small tumors
c. Liquid based pap smear is done d. Conization done first prior to the
annually chemoradiation
d. HPV test every 2 years
8. A 55 y/o multigravida is diagnosed with cervical
3. A patient underwent colposcopic-guided cervical cancer. On pelvic exam, the cervix is nodular, 7x6
punch biopsy. Histopathologic examination showed cm, extending up to the lower 3rd of the vagina,
undifferentiated or immature squamous cells bilateral parametria nodular. The CT scan result
confine to the lower 3rd of the epithelium. How will showed no lymph node involvement. What is the
you manage the patient? stage of the disease?
a. Co-testing after 1 year a. IIA2
b. Immediate HPV testing b. IIB
c. Repeat Colposcopy c. IIIA
d. Conization d. IIIB

4. Which of the following is true about the 9. A patient underwent conization for CIN III.
transformation zone? Histopathologic examination showed malignant
a. Corresponds to the squamocolumnar squamous cells occupying the cervical epithelium
junction with 5 mm invasion to the cervical stroma. What is
b. Line by columnar epithelium the stage of the disease?
c. Area outside the outer old squamocolumnar a. Carcinoma in situ
junction b. IA1
d. Area of squamous metaplasia c. IA2
d. IB1
5. In HPV infection, which of the following viral
factors are necessary for cell immortalization? 10. The CT scan result of a patient newly
a. E1 and E2 diagnosed with stage IIIB cervical cancer showed a
b. E4 and E5 bladder mass measuring 2x1 cm. Which of the
c. E6 and E7 following is correct regarding the evaluation and
d. L1 and L2 management of this case?
a. Change the stage to IVA
6. CERVICAL CANCER b. Advise cystoscopy and biopsy of the
A 45 y/o multigravida came in for post-coital bladder mass
bleeding. On pelvic exam, a 0.5 cm mass was c. Request for an MRI
noted at the 3 o’clock position of the cervix. What is d. Give systemic chemotherapy immediately
your next step in the management?
a. Perform cervical cytology 11. ENDOMETRIAL CANCER
b. Colposcopic-guided biopsy Which of the following patients carries the
c. Cervical punch biopsy? worse prognosis?
d. Observe a. 60 y/o nulligravid, normal BMI, with stage
IIIA, grade 1 endometrioid adenocarcinoma
7. Pelvic exam on a cervical patient revealed a 1x1 b. 40 y/o G1P1, obese, with stage II, grade 2
nodular mass at the anterior lip of the cervix, endometrioid adenocarcinoma
smooth vagina and free parametria. Which of the c. 55 y/o G3P3, normal BMI, with stage IB,
following is true regarding the management of this serous papillary carcinoma of the
case? endometrium

GYNE TRANS TEAM Page 4 of 10


GYNECOLOGY SAMPLEX
4th Bimonthly

d. 65 y/o, nulligravid, overweight, with stage


IA, grade 3 endometrioid adenocarcinoma 16. OVARIAN CANCER
A 42 y/o G1P1 came in for sudden abdominal
12. A 55 y/o G1P1 underwent endometrial biopsy enlargement, ultrasound was requested. Which of
for biopsy for postmenopausal bleeding with the following findings will help you rule-out a
thickened endometrium. Histopathology revealed possible ovarian malignancy?
endometrioid adenocarcinoma of the endometrium. a. Cystic, uniloculated adnexal mass?
What is the management of choice? b. Presence of papillary excrescences
a. Radical Hysterectomy with Bilateral c. Peritoneal implants
Salpingo-oophorectomy, Bilateral Lymph d. Presence of ascites
Node Dissection, Para-aortic Lymph Node
Sampling 17. On histologic examination, the ovarian mass
b. Extrafascial Hysterectomy with Bilateral showed epithelial cells resembling those of the
Salpingo-oophorectomy, Bilateral Lymph endocervix. What is your epithelial tumor
Node Dissection, Para-aortic Lymph diagnosis?
Node Sampling a. Serous
c. TAHBSO, Peritoneal Fluid Cytology, BLND, b. Mucinous
PALS, Random Peritoneal Sampling, c. Endometrioid
Omentectomy d. Clear Cell
d. External Beam Radiation Therapy followed
by Brachytherapy 18. A 75 y/o G4P4 underwent surgery for ovarian
malignancy. Intra-operatively, a ruptured ovarian
13. A patient underwent surgery for endometrial mass was noted and the rest of the pelvoabdominal
malignancy. Gross and histologic examination of organs were unremarkable. What is the stage of
the specimen showed and endometrial mass the disease?
invading >50% of the myometrium, the tumor. a. IA
involvement of the right fallopian tube, the rest of b. IC1
the organs were unremarkable. What is the stage c. IC2
of the disease? d. IC3
a. IA
b. IB 19. A 45 y/o G0 underwent surgery for ovarian new
c. IIIA growth. Final histopathology confirmed ovarian
d. IIIB malignancy with microscopic metastatic of the
pelvic peritoneum. What is the stage of the
14. Histologic examination of an endometrioid disease?
adenocarcinoma specimen showed less than 5% of a. IIA
non-glandular or solid tumor growth pattern. How b. IIB
will you grade this findings? c. IIIA1
a. Grade 0 d. IIIA2
b. Grade 1
c. Grade 2 20. What gene mutation is associated with 39-54%
d. Grade 3 of lifetime risk of ovarian cancer
a. BRCA1
15. A 46 y/o nulligravid underwent surgery for b. BRCA2
endometrial cancer. Intraoperatively there was a c. Lynch II
note of 2x2 cm tumor implant of the omentum. d. pTEN
Examination of the uterus revealed and endometrial
mass with >50% myometrial invasion with cervical
stromal involvement. What is the stage of the
disease?
a. IB
b. II
c. IIIB
d. IVB
GYNE TRANS TEAM Page 5 of 10
GYNECOLOGY SAMPLEX
4th Bimonthly

QUIZ 8: AMENORRHEA,
HYPERPROLACTINEMIA

1. Primary amenorrhea is most commonly due to a. Ovary


this condition b. Adrenals?
a. Testicular feminization c. Leydig cells
b. Roki Tansky Kuster Hauser syndrome d. Granulosa cells
c. Utero-vaginal agenesis
d. Gonadal failure 9. Roki Tansky-Kuster-Hauser Syndrome is an
isolated developmental defect that is genetically
2. This secretes anti mullerian hormone affecting inherited characterized by
sexual development of the individual a. Breast development, uterus present
a. Testes b. Absent breast development, uterus absent
b. Leydig cells c. Absent breast development, uterus present
c. H-Y antigen d. Breast development, uterus absent?
d. Sertoli cells
10. The last sign of puberty
3. This is considered as delayed menarche when a. Development of pelvic contour
onset of menstruation occurs at this age, in the b. Appearance of pubic hair
absence of an abnormality c. Menarche
a. 15 years old d. Rapid growth rate
b. 18 years old?
c. 16 years old? 11. The most important and probably most common
d. 19 years old cause of amenorrhea in adolescent girls
a. stress?
4. Secondary amenorrhea occurs in this condition b. anorexia nervosa
a. Premature ovarian follicle? c. exercise
b. Gonadal failure d. hormonal changes
c. Roki Tansky Kuster Hauser syndrome
d. Kallman syndrome 12. Prolactin release is mainly inhibited by a
neurotransmitter
5. This structure is not derived from the Mullerian a. Serotonin
duct b. Dopamine
a. Ovary c. Norepinephrine
b. Fallopian tube d. Indolamine
c. Uterus
d. Cervix 13. In order to avoid a falsely elevated result for
Prolactin test, sample collection is done
6. Delayed menarche least likely occurs among a. After a meal
a. Swimmers b. When the patient just woke up
b. Ballet dancers c. After an exercise
c. Pianists d. When the patient is relaxed
d. Runners
14. Galactorrhea due to hyperprolactinemia occurs
7. During puberty, breast budding is followed by due to this mechanism
a. Development of pelvic contour a. elevated estrogen level causing increased
b. Appearance of pubic hair secretion and release of Prolactin on the
c. Menarche breast
d. Rapid growth rate b. elevated Thyroid releasing hormone which
stimulates Prolactin release on the breast
8. During puberty, this is mainly involved mainly in c. estrogen alters the secretion of GnRH from
the secretion of substance responsible for the the hypothalamus affecting Prolactin
development of sexual hair release on the breast

GYNE TRANS TEAM Page 6 of 10


GYNECOLOGY SAMPLEX
4th Bimonthly

d. low estrogen level causes Prolactin to act d. monomeric forms


on the receptors on the breast tissue
22. Among the medications that cause
15. The most common cause for Galactorrhea hyperprolactinemia, this does not cause
a. Pregnancy galactorrhea
b. Prolactinoma a. Antihypertensive
c. Ingestion of medications b. Anxiolytics
d. Stress c. Oral contraceptive pills
d. Antiemetics
16. Physiologic hyperprolactinemia is commonly
caused by 23. The most common pituitary tumor associated
a. Sleep with Hyperprolactinemia
b. Exercise a. Lactotroph hyperplasia
c. Buffet banquet b. Prolactinoma
d. Stress c. Empty sella syndrome
d. Functional hyperprolactinemia
17. Prolactin as a polypeptide is best describe as
a. secreted by the chromophobe cells at the 24. This condition does not cause
anterior area of the pituitary gland Hyperprolactinemia
b. secreted by the lactotrophs upon stimulation a. Chronic kidney disease
by the GnRH from the Hypothalmus b. Hyperthyroidism
c. secreted and stored by the lactotrophs c. Liver cirrhosis
on the lateral areas of the pituitary gland d. Cushing syndrome
d. secreted and released by the chromophobe
cells from the pituitary gland 25. This is the most common form of
Cryptomenorrhea
18. When hyperprolactinemia occurs in women of a. Transverse vaginal septum?
reproductive age, this may result in b. Imperforate hymen
a. menorrhagia c. Adhesive vaginitis
b. oligomenorrhea d. Cervical stenosis
c. hypermenorrhea
d. iatrogenic menometrorrhagia QUIZ 9: HYPERANDROGENISM, PCOS, &
FERTILITY
19. Effects of Hyperprolactinemia on the Infertility Dr. Lita Set 1
hypothalamic-pituitary ovarian axis
a. The episodic release of Follicle stimulating 1. This is a major reason for infertility in women
hormone is reduced a. Anovulation
b. There is reduced pulsatility of the GnRH b. Weight
c. The amplitude of LH secretion is reduced c. Age
d. The episodic release of GnRH becomes d. All of these
reduced
2. Boosting the intake of _________ can help
20. Bromocriptine as an ergot alkaloid used for women trying to conceive
hyperprolactinemia is considered as a. Potassium
a. dopamine receptor antagonist b. Folic acid
b. dopamine receptor agonist c. Lycopene
c. blocks Dopamine uptake d. Nickel
d. inhibits the conversion of Tyrosin to dopa
3. Nerve supply of ovary is through
21. The biologically active Prolactin is characterized a. Sacral fibers
as b. Pudendal nerve
a. the larger forms c. Branches of renal and aortic plexus
b. polymeric forms d. none of the above
c. the aggregation of monomeric
GYNE TRANS TEAM Page 2 of 10
GYNECOLOGY SAMPLEX
4th Bimonthly

ovaries on ultrasound, menstrual


4. Menstrual phase bleeding is due to irregularity
a. necrosin d. chronic anovulation, hyperandrogenemia,
b. fibrinolysis elevated LH levels
c. menotoxin
d. vasoconstriction and ischaemia 2. Preferred screening test for glucose intolerance
or Type 2 diabetes mellitus in women with PCOS
5. Fertility in women is impaired by a. fasting blood sugar
a. Hernia b. 75-gram oral glucose tolerance test
b. High blood pressure c. HbA1c
c. Obesity d. random blood sugar
d. Gallstones
3. False regarding hormone levels in polycystic
6. Infertility is when a couple fails to conceive after ovarian disease?
a. 4 years of unprotected sex a. Increased androgen
b. 3years of unprotected sex b. Increased insulin
c. 2 years of unprotected sex c. Increased FSH
d. 1 years of unprotected sex d. Increased LH

7. Procedures used to treat infertility are 4. One of the following is not indicative of
a. ICSI – Intracytoplasmic Sperm Injection hyperandrogenism
b. IVF – In Vitro Fertilization a. increased testosterone levels
c. IUI – Intrauterine insemination b. increased sex hormone binding globulin
d. All of these c. increased dehydroepiandrosterone
d. increased androstenedione
8. Right ovarian artery arises from
a. Uterine artery 5. All of the following EXCEPT one describes a
b. Aorta woman with Phenotype D PCOS
c. Renal artery a. with mild PCOS
d. Common iliac artery b. clinical or biochemical
hyperandrogenism
9.This condition impair infertility c. with polycystic ovaries
a. PCO d. anovulation
b. Fibroids
c. Endometriosis 6. Determination for serum 17-
d. All of these hydroxyprogesterone level is requested to rule out
a. Cushing syndrome
10. In women, this sexually transmitted disease can b. Congenital adrenal hyperplasia
result in infertility c. Insulin resistance
a. Human papillomavirus (HPV) d. Androgen-producing ovarian tumor
b. Pelvic inflammatory disease
c. Genital herpes 7. Which of the following is NOT true about sex-
d. None of the above hormone binding globulin (SHBG)?
a. It binds to most of the circulating
PCOS Dr. Vilches Set 2 testosterone.
1. The Rotterdam criteria in the diagnosis b. It is produced by the liver
of Polycystic Ovarian Syndrome consist of the c. It is increased when there is
following? hyperinsulinemia.
a. anovulation, increased LH/FSH ratio, d. It is increased by estrogen
ultrasound finding of polycystic ovaries
b. anovulation, insulin resistance, clinical signs 8. The drug presently recommended by the
of androgen excess American College of Obstetricians and
c. clinical or biochemical evidence of Gynecologists (ACOG) for ovulation induction in
androgen excess, finding of polycystic
GYNE TRANS TEAM Page 3 of 10
GYNECOLOGY SAMPLEX
4th Bimonthly

patients with PCOS because of its high pregnancy a. legs


and birth rates compared to clomiphene citrate b. armpit
a. metformin c. chin
b. letrozole d. pubis
c. spironolactone
d. GnRH agonist 6. The resting phase of hair growth such that the
follicle does not go beyond the dermis
9. Which enzyme catalyzes peripheral conversion a. Exogen
of testosterone to the more potent b. Catagen
dihydrotestosterone? c. Telogen
a. 17ß-hydroxysteroid dehydrogenase d. Anagen
b. 5α-reductase
c. cytochrome P450 aromatase 7. Virilization occurs when there is a markedly
d. 3-alpha-G-diol elevated circulating levels of
a. andostenedione
10. The following are the consequences of b. testosterone
hyperandrogenism, EXCEPT c. dehydroepiandrosterone
a. acanthosis nigricans d. dehydroepiandrosterone sulfate
b. anovulation
c. hirsutism, acne or androgenic alopecia 8. When the testosterone level is too high, the
d. arrest in antral follicle development organ of pathology is in the
a. ovary
Hyperandrogenism Dr. Barrot Set 3 b. pilosebaceous unit
1. If there is an abnormality of the pilary unit, this c. adrenal gland
may manifest as d. peripheral tissues
a. alopecia
b. hypertrichosis 9. Dehydrepiandrosterone sulfate is mainly
c. acne produced by the
d. polycystic ovarian syndrome a. ovary
b. pilosebaceous unit
2. This is the main androgen produced by the ovary c. adrenal gland
a. testosterone d. peripheral tissues
b. androstenedione
c. dehydroepiandrosterone sulphate 10. When treating for hirsutism, the effect may be
d. dehydroepiandrosterone noticed
a. after 3 months of therapy
3. The skin manifestation that first responds when b. after 1 month of therapy
treating hyperandrogenism c. after 6 weeks of therapy
a. hirsutism d. after 8 weeks of therapy
b. alopecia
c. acne 11. Characteristic for testosterone is that most of
d. breast atrophy the circulating levels are
a. tightly bound to the albumin
4. A 27 year old office worker seeks consult b. loosely bound to the sex hormone binding
because of virilization. Her testosterone level was globulin
markedly elevated but her dehydroepiandosterone c. biologically inactive
sulfate level was normal The most likely pathology d. tightly bound to sex hormone binding
is in the albumin
a. Ovary
b. adrenals 12. The most common condition during the
c. pilosebaceous unit reproductive period associated with Hirsutism
d. periphery a. Pregnancy luteoma
b. Polycystic Ovarian Syndrome
5. Vellus hair are mainly found on c. Adult-onset Congenital adrenal hyperplasia
GYNE TRANS TEAM Page 4 of 10
GYNECOLOGY SAMPLEX
4th Bimonthly

d. Adrenal tumor a. chin


b. below the breast
13. The skin disorder caused by hyperandrogenism c. hips
that is least responsive to therapy d. palms
a. acne
b. alopecia
c. hirsutism
d. hypertrichosis

14. Abnormalities of the sebaceous component


a. alopecia
b. hypertrichosis
c. acne
d. polycystic ovarian syndrome

15. When this phase of hair growth is prolonged,


the hair becomes thicker.
a. Exogen
b. Catagen
c. Telogen
d. Anagen

16. A 32 year old woman presented with hirsutism.


Her laboratory parameters were normal except for
markedly elevated 3αdiol-G. This abnormal result
indicates that that the organ of pathology is in the
a. ovary
b. adrenals
c. kidney
d. periphery

17. A 28 year old G2P1 at 28 weeks came in


because her ultrasound result for fetal aging
revealed an incidental finding of bilaterally cystic,
ovarian enlargement. The patient also presents
with mild hirsutism. The patient most likely has a
concomitant
a. polycystic ovarian disease
b. stromal hyperthecosis
c. hyperreactio luteinalis
d. adrenal tumor

18. Sertoli-Leydig cell tumors cause a markedly


elevated testosterone levels that is
a. more than two and a half times the upper
limits of the normal range
b. more than ½ of the upper limit of normal
range
c. doubling of the upper limit of normal value
d. two times the upper limit of normal value

19. The structure in the human body that is devoid


of the pilosebaceous unit

GYNE TRANS TEAM Page 5 of 10


GYNECOLOGY SAMPLEX FOURTH BIMONTHLY
S.Y. 2020-2021 | DRAVERGONZ 2022

1. Where does fertilization normally occur? She is confirmed as pregnant if there is


A. Uterus __________ in her urine.
B. Ovary A. Hyaluronidase
C. Vagina B. Luteinizing hormone
D. Fallopian tube C. Follicle stimulating hormone
D. Human chorionic gonadotropin
2. The ovum can be fertilized for about how
many hours after ovulation? 9. Which of the following represents the correct
A. 5-10 order of prenatal development?
B. 24-48 A. Embryonic, fetal, germinal
C. 4-20 B. Fetal, embryonic, germinal
D. 1-3 C. Germinal, embryonic, fetal
D. Fetal, germinal, embryonic
3. Which of the following is NOT a method for
optimizing the chances of conception? 10. In the developing fetus, the head takes
A. Engaging in intercourse within a few precedence over the growth of the lower parts
hours of ovulation due to
B. Checking blood samples for A. Nagele’s rule
testosterone levels B. The proximodistal principle
C. Analyzing the woman’s urine or saliva C. The cephalocaudal principle
for LH D. The embryonic disk
D. Charting basal body temperature
11. What’s the mechanism of action of
4. Which of the following is typically NOT a finasteride in the treatment of hirsutism?
cause of male fertility problems? A. competes with androgens for androgen
A. Genetics receptors
B. Pituitary imbalance B. blocks conversion of testosterone to
C. Irregularly shaped sperm dihydrotestosterone
D. Irregularly shaped sperm C. decreases LH pulsatility and amplitude
D. improves insulin resistance
5. The major causes of infertility in females
include all of the following, EXCEPT 12. The best medical treatment of signs and
A. Failure to ovulate symptoms of PCOS in adolescents
B. Obstructions of the reproductive tract A. metformin
C. Endometriosis B. oral contraceptives
D. An irregularly shaped uterus C. ovarian drilling
D. GnRH analogues
6. Charley and Kirsten have been trying to have
a baby for over a year. They just found out that 13. Which of the following statements is NOT
Charley has a very low sperm count. What true?
method are they candidates for? A. Risk for endometrial hyperplasia or
A. Surrogate fatherhood malignancy is increased with chronic
B. In vitro fertilization anovulation.
C. Donor IVF B. Use of oral contraceptives for at least
D. Artificial insemination 2 years can lower risk of ovarian and
endometrial cancers.
7. Bailey and Sarah have been trying to have a C. As woman ages, PCOS phenotype may
baby for over a year. They just found out that change or disappear.
Sarah’s ova are viable, but she is not ovulating D. Ultrasound is not recommended in the
on a regular basis. What fertility method are they diagnosis of PCOS in adolescents within
candidates for? 8 years of menarche.
A. Donor IVF
B. Adoption 14. In the treatment of PCOS with combined oral
C. Embryonic transfer contraceptives, the estrogen component has
D. In vitro fertilization which of the following effects?
A. Reduces luteinizing hormone production
8. Julia missed her period about three weeks B. Reverses endometrial hyperplasia
ago and asks her physician for a pregnancy test. C. Increases 5α-reductase activity

GYNE Trans Team Page 1 of 5


GYNECOLOGY SAMPLEX FOURTH BIMONTHLY
S.Y. 2020-2021 | DRAVERGONZ 2022

D. Increases sex hormone-binding B. abortion


globulin production C. blood dyscrasia
D. diabetes
15. Ultrasound findings of the ovaries in PCOS,
except 22. The part of the endometrium that
A. ovarian volume of >10cu mm regenerates after menstruation
B. presence of at least 10 cystic follicles A. zona basale
in either ovary B. zona functionale
C. presence of cystic follicles about 2-8 C. zona spongiosa
mm in diameter D. zona compacta
D. peripherally-located cystic follicles about
2-8mm in diameter 23. The first line diagnostic tool for assessing
structural abnormality with heavy menstrual
16. Which of the following medications can be bleeding
added to improve the clomiphene citrate A. pelvic exam
response in women with PCOS? B. pap smear
A. Dehydroepiandrosterone C. ultrasound
B. Insulin D. CT scan
C. Metformin
D. Progesterone 24. The Primary cause to be considered as a
cause of AUB in the reproductive age group
17. Which of the following conditions is a long- A. polyp
term consequence of a woman with PCOS? B. blood dyscrasia
A. cervical cancer C. pregnancy
B. osteoporosis D. malignancy
C. Cushing syndrome
D. Type 2 diabetes mellitus 25. The first mechanism for hemostasis during
menstruation
18. If you suspect presence of A. platelet plug formation
hyperandrogenemia, which of the following B. vasoconstriction
would you request for? C. uterine contraction
A. DHEA-S D. coagulation
B. total testosterone
C. free androgen index 26. Menstrual bleeding is light
D. androstenedione A. < 5 ml
B. 5-10 ml
19. Based on the 2018 guidelines by ESHRE C. 10 – 30 ml
and ASRM, which of the following regarding D. 30-35 ml
diagnosis of PCOS in adolescents is TRUE?
A. ultrasound is recommended if patient is 27. Classification of abnormal uterine bleeding
at least 3 years postmenarche commonly seen during the pubertal stage
B. all of the 3 Rotterdam criteria should be A. coagulopathy
present B. av malformation
C. evidence of hyperandrogenemia C. iatrogenic
should be present and not just the D. malignancy
clinical signs
D. ovarian volume measurement on 28. Classification of abnormal uterine bleeding
ultrasound is not necessary due to intake of contraceptive pills
A. coagulopathy
20. First-line treatment for PCOS B. ovulatory dysfunction
A. ovulation induction C. iatrogenic
B. gonadotropins? D. malignancy
C. in-vitro fertilization
D. ovarian diathermy on drilling 29. The blood supply to the endometrium is from
A. Arcuate artery
21. The common cause for abnormal uterine B. Spiral artery
bleeding during the postmenstrual and C. Radial artery
premenstrual years D. Basal artery
A. anovulation

GYNE Trans Team Page 2 of 5


GYNECOLOGY SAMPLEX FOURTH BIMONTHLY
S.Y. 2020-2021 | DRAVERGONZ 2022

30. This is a structural cause for abnormal


uterine bleeding in a 20 year old G2P2 37. This is not a common affective symptom
A. Polyp among patients with Premenstrual syndrome
B. Von Willebrand’s disease A. irritability
C. Dengue Fever B. depression
D. Levonorgestrel containing intrauterine C. labile mood
system D. perception of weight gain

31. A 40 year old G0 came in because of vaginal 38. The laboratory test that is performed in
bleeding for 2 weeks. Cervix and ovaries were coming up with a diagnosis of PMDD
normal. Ultrasound revealed a thickened A. estrogen level
endometrium. You assess her as B. FSH level
A. AUB-P C. ultrasound
B. AUB-M D. none available
C. AUB-A
D. AUB-L 39. This has not been proven to be a cause for
PMS
32.a 15 year old was brought in because of A. hormonal imbalance
prolonged menses for 3 weeks. Her menarche B. vitamin deficiency
was at 11 years old for 3 days. Subsequent C. hormonal allergy
menses were unremarkable. She has occasional D. excess prolactin
gum bleeding and epistaxis. The pelvic
structures were normal except for moderate 40. The appearance of pubic hair follows
bleeding seen through the introitus. You A. breast budding
consider B. widened pelvic contour
A. AUB-I C. rapid growth spurt
B. AUB-C D. menarche
C. AUB-P
D. AUB-O 41. This is the first sign that the girl has entered
the stage of puberty
33. A woman experiencing symptoms during her A. breast budding
second half of menstrual cycle has intact B. wide pelvic contour
cognitive function and performance despite C. appearance of pubic hair
having this condition. D. menarche
A. Premenstrual syndrome
B. Premenstrual dysphoric disorder 42. The structure that is not derived from the
C. Heavy menstrual bleeding mullerian duct:
D. Abnormal uterine bleeding A. ovary
B. fallopian tube
34.The most common symptom noted among C. uterus
clients with Premenstrual dysphoric disorder D. upper 3rd of vagina
A. depression
B. hebephrenic behaviour 43. The development of the gonadal sex occurs
C. panic attacks during this period
D. anxiety A. upon fertilization
B. once the ovary develops
35. The neurohormone that is highly associated C. upon secretion of the mullerian
with premenstrual dysphoric disorder hormone
A. bromocriptine D. once there is the conversion of
B. dopamine testosterone to dihydrotestosterone
C. prolactin
D. serotonin 44. The appearance of this manifestation
reflects the last stage of puberty
36. This condition greatly impairs the woman’s A. breast budding
personal functioning B. wide pelvic contour
A. Premenstrual syndrome C. appearance of pubic hair
B. Premenstrual dysphoric disorder D. menarche
C. Heavy menstrual bleeding
D. Abnormal uterine bleeding

GYNE Trans Team Page 3 of 5


GYNECOLOGY SAMPLEX FOURTH BIMONTHLY
S.Y. 2020-2021 | DRAVERGONZ 2022

45. The genetic sex is determined during this C. she has secondary amenorrhea
period before 40 years old
A. upon fertilization D. she has amenorrhea at 48 years old
B. once the ovary develops after undergoing salpingooophorectomy
C. upon secretion of the mullerian hormone
D. once there is the conversion of 52. An 18 year old sought consult because of
testosterone to dihydrotestosterone severe dysmenorrhea. She is unable to report
for work and been in conflict with her employers.
46. The onset of menstruation will be early Her menarche was 13 years old for 3 days,
among this group, (except): associated with dysmenorrhea. The findings on
A. ballet dancers the pelvis were unremarkable. This is a case
B. swimmers A. Secondary dysmenorrhea
C. athletes B. Primary Dysmenorrhea
D. pianists C. Endometriosis
D. PMDD?
47. During puberty, rapid growth spurt precedes
this developmental change 53. A 26 year old G0 presents with severe
A. breast budding dysmenorrhea. Her pelvic structures are
B. widened pelvic contour apparently normal. You attribute the cause for
C. appearance of pubic hair her complaint to:
D. menarche A. obstruction of the menstrual outflow
B. retrograde flow of menstrual fluid
48. Secondary amenorrhea on a 28 year old G0 C. endogenous prostaglandin levels
is least likely to occur from this condition D. adhesions from previous infections
A. after bilateral cystectomy
B. irradiation 54. The treatment of choice for a 25 year old G0
C. chemotherapy married for 5 months complaining of
D. asherman’s syndrome dysmenorrhea with normal pelvic findings.
A. Mefenamic acid
49. This has been observed before onset of B. Transcutaneous electric nerve
puberty stimulation since it improves the mood
A. The hypothalamus is sensitive to the C. combined OCP since it promotes good
positive effect of low estrogen levels patient compliance
B. The hypothalamus is sensitive to the D. Levonorgestrel releasing intrauterine
negative feedback of low estrogen system because it reduces the pain as
levels much as 90%
C. The pituitary gland is insensitive to the
negative feedback of the hypothalamus 55. Secondary dysmenorrhea is not attributed to
D. The pituitary gland does not to secrete this condition
gonadotropic hormones causing low A. Pelvic inflammatory disease
estrogen B. heterotopic pregnancy
C. presence of leiomyoma
50. A 26 year old went to the clinic because of D. presence of functional bowel disease
primary amenorrhea. If you are considering Roki
Tansky-Kuster-Hauser Syndrome you will 56. Secretion of Prolactin is a function of the
observe the following A. hypothalamus
A. breast exam showed no breast budding B. pituitary gland
B. she is of short stature for her age C. amygdala
C. pelvic examination reveals a short or D. breast
absent vagina
D. the ovaries are absent on transvaginal 57. This is considered the most common cause
ultrasound examination for physiologic hyperprolactinemia
A. sleep
51. An individual is considered to have a B. exercise
premature ovarian failure C. stress
A. she has secondary amenorrhea at 45 D. breast stimulation
years old
B. she has primary amenorrhea at 25 58. The biologically active form of Prolactin is
years old characterized by

GYNE Trans Team Page 4 of 5


GYNECOLOGY SAMPLEX FOURTH BIMONTHLY
S.Y. 2020-2021 | DRAVERGONZ 2022

A. being larger in molecular size 66. When the testosterone level is too high,
B. produced in small quantity but with consider a pathology of the
significant effects A. Adrenal gland
C. an aggregation of monomers B. Ovary
D. monomeric forms C. Liver
D. Renal
59. The main symptom with hyperprolactinemia
during the non-pregnant state 67. Results from an abnormality in the pilary
A. anovulation component of the pilosebaceous unit
B. AUB A. Pimple
C. oligomenorrhea B. Black heads
D. galactorrhea C. Acne
D. Alopecia
60. The secretion of Prolactin is mainly inhibited
at the Hypothalamic level by 68. Characteristic with virilization
A. Serotonin A. Rapid onset
B. Thyrotropin B. Breasts enlarge
C. Bromocriptine C. Clitoris recedes
D. Dopamine D. Acne

61. Hyperprolactinemia and Thyroid disorder 69. The resting phase of hair growth such that
A. Hypothyroidism increases Prolactin the follicle does not go beyond the dermis
secretion by the TRH A. Anagen
B. Hypothyroidism increases Prolactin B. Catagen
secretion by the TSH C. Telogen
C. Hyperthyroidism increases Prolactin D. Exogen
release due to increased TSH secretion
D. Hyperthyroidism causes stimulation of 70. The manifestation that first responds when
Prolactin secretion by the Thyroid treating hyperandrogenism
releasing hormones A. Hirsutism
B. Acne
62. Sample collection for Prolactin levels is not C. Alopecia
done D. Breast atrophy
A. before an exercise
B. after attending a party
C. before taking an exam
D. before siesta time

63. Galactorrhea is diagnosed by


A. Ultrasound of the breast
B. CT scan of the breast
C. MRI of the breast
D. microscopic examination of breast
secretion

64. The main androgen produced by the ovary


A. testosterone
B. androstenedione
C. dehydroepiandosterone
D. DHEAS

65. The main androgen produced by the


adrenal gland
A. Testosterone
B. Androstenedione
C. Dehydroepiandrosterone
D. DHEAS

GYNE Trans Team Page 5 of 5

You might also like